SlideShare a Scribd company logo
1 of 90
Download to read offline
−1/2   1
                                                                 f (x0 ) = f (0) = (2 · 0 + 9)      =
                                                                                                      3
                                                                 So,
                                                                 L(x) = f (x0 ) + f (x0 ) (x − x0 )
                                                                              1
                                                                       = 3 + (x − 0)
                                                                              3
Chapter 3                                                              =3+ x
                                                                              1
                                                                              3
                                                             (b) Using √ the approximation L(x) to esti-
                                                                                       √
                                                                 mate 8.8, we get 8.8 = f (−0.1) ≈
Applications of                                                                   1
                                                                 L(−0.1) = 3 + (−0.1) = 3 − 0.033 =
                                                                                  3
                                                                 2.967
Differentiation                                           4. (a) f (x) =
                                                                          2
                                                                            , x0 = 1
                                                                          x
                                                                 f (x0 ) = f (1) = 2
                                                                              2
                                                                 f (x) = − 2 and so f (1) = −2
                                                                             x
3.1   Linear Approximations                                      The linear approximation is
                                                                 L(x) = 2 + (−2) (x − 1)
      and Newtons Method
                 √                                           (b) Using the approximation L(x) to estimate
 1. (a) f (x) = x, x0 = 1                                          2            2
                         √                                            , we get      = f (0.99) ≈ L(0.99) =
        f (x0 ) = f (1) = 1 = 1                                  0.99          0.99
                  1                                              2 + (−2)(0.99 − 1) = 2.02
        f (x) = x−1/2
                  2
                          1                              5. (a) f (x) = sin 3x, x0 = 0
        f (x0 ) = f (1) =                                       f (x0 ) = sin(3 · 0) = 0
                          2
        So,                                                     f (x) = 3 cos 3x
        L(x) = f (x0 ) + f (x0 ) (x − x0 )                      f (x0 ) = f (0) = 3 cos(3 · 0) = 3
                      1                                         So,
               = 1 + (x − 1)                                    L(x) = f (x0 ) + f (x0 ) (x − x0 )
                      2
                  1 1                                                  = 0 + 3 (x − 0)
               = + x
                  2 2                                                  = 3x
    (b) √
        Using the approximation L(x) to estimate
                        √                                    (b) Using the approximation L(x) to esti-
           1.2, we get 1.2 = f (1.2) ≈ L(1.2) =
        1 1                                                      mate sin(0.3), we get sin(0.3) = f (0.1) ≈
           + (1.2) = 1.1                                         L(0.1) = 3(0.1) = 0.3
        2 2
 2. (a) f (x0 ) = f (0) = 1 and                          6. (a) f (x) = sin x, x0 = π
                  1        −2/3
                                                                f (x0 ) = sin π = 0
        f (x) = (x + 1)
                  3                                             f (x) = cos x
                      1
        So, f (0) =                                             f (x0 ) = f (π) = cos π = −1
                      3                                         The linear approximation is,
        The Linear approximation is,
                      1              1                          L(x) = f (x0 ) + f (x0 ) (x − x0 )
        L(x) = 1 + (x − 0) = 1 + x
                      3              3                                 = 0 + (−1) (x − π) = π − x
    (b) Using the approximation L(x) to estimate
        √               √
         3               3
                                                            (b) Using the approximation L(x) to esti-
           1.2, we get 1.2 = f (0.2) ≈ L(0.2) =                 mate sin(3.0), we get sin(3.0) = f (3.0) ≈
             1
        1 + (0.2) = 1.066                                       L(3.0) = π − 3.0
             3
                 √                                                      √
                                                                        4
 3. (a) f (x) = 2x + 9, √0 = 0
                           x                             7. (a) f (x) = √ 16 + x, x0 = 0
                                                                        4
        f (x0 ) = f (0) = 2 · 0+9 = 3                           f (0) = 16 + 0 = 2
                   1                                                     1
        f (x) = (2x + 9)
                             −1/2
                                  ·2                            f (x) = (16 + x)−3/4
                   2                                                     4
                                                                        1                 1
                = (2x + 9)
                           −1/2                                 f (0) = (16 + 0)−3/4 =
                                                                        4                 32

                                                   150
3.1. LINEAR APPROXIMATIONS AND NEWTONS METHOD                                                        151

        L(x) = f (0) + f (0)(x − 0)                                           36
                                                               L(72) = 120 +     (72 − 80)
                     1                                                        20
             =2+ x                                                    = 120 + 1.8(−8)
                    32
                     1                                                = 105.6 cans
             = 2 + (0.04) = 2.00125
                    32                                                         168 − 120
                       1                                   (b) L(x) = f (100) +          (x − 100)
    (b) L(0.08) = 2 + (0.08) = 2.0025                                           100 − 80
                       32                                                    48
                       1                                       L(94) = 168 − (94 − 100)
    (c) L(0.16) = 2 + (0.16) = 2.005                                         20
                       32                                            = 168 − 2.4(−6)
 8. (a) f (x) = sin x, x0 = 0                                         = 182.4 cans
        f (0) = 0
        f (x) = cos x                                                          142 − 128
        f (0) = cos 0 = 1                             11. (a) L(x) = f (200) +           (x − 200)
                                                                               220 − 200
        L(x) = f (0) + f (0) (x − 0)                                          14
                                                               L(208) = 128 + (208 − 200)
             =0+1·x                                                           20
        L(0.1) = 0.1                                                  = 128 + 0.7(8) = 133.6
    (b) f (x) = sin x, x0 = π                                                  142 − 136
                   √           3
            π        3                                     (b) L(x) = f (240) +          (x − 240)
        f       =                                                              220 − 240
            3       2                                                          6
             π           π    1                                L(232) = 136 − (232 − 240)
        f        = cos =                                                      20
             3           3    2                                       = 136 − 0.3(−8) = 138.4
                     π            π         π
        L(x) = f          +f            x−
                 √ 3              3         3
                   3 1            π                                                14 − 8
        L(1) =       +       1−        ≈ 0.842        12. (a) L(x) = f (10) +             (x − 10)
                  2      2         3                                               10 − 5
                              2π                                            6
    (c) f (x) = sin x, x0 =                                    L(8) = 14 + (−2) = 11.6
                     √          3                                           5
            2π          3                                                      14 − 8
        f         =                                        (b) L(x) = f (10) +        (x − 10)
             3         2                                                       10 − 5
             2π            2π        1                                       6
        f          = cos      =−                               L(12) = 14 + (2) = 16.4
              3             3        2                                       5
                      2π             2π         2π
        L(x) = f            +f             x−         13. f (x) = x3 + 3x2 − 1 = 0, x0 = 1
                       3              3          3
                 √                                        f (x) = 3x2 + 6x
                    3 1             2π
               =      −       x−
                  2√ 2               3
                                                                          f (x0 )
            9         3 1 9 2π                             (a) x1 = x0 −
        L        =       −          −       ≈ 0.788                       f (x0 )
            4        2      2 4         3
                                                                        13 + 3 · 12 − 1
                          18 − 14                                 =1−
 9. (a) L(x) = f (20) +           (x − 20)                               3 · 12 + 6 · 1
                          20 − 30                                       3     2
                        4                                         =1− =
         L(24) ≈ 18 −     (24 − 20)                                     9     3
                       10                                                 f (x1 )
                = 18 − 0.4(4)                                  x2 = x1 −
                                                                          f (x1 )
                = 16.4 games                                                2 3          2 2
                                                                      2     3     +3     3     −1
                         14 − 12                                  =     −
                                                                      3           2 2          2
    (b) L(x) = f (40) +          (x − 40)                                   3     3     +6     3
                         30 − 40
                       2                                               79
        f (36) ≈ 12 − (36 − 40)                                   =       ≈ 0.5486
                      10                                              144
               = 12 − 0.2(−4)                              (b) 0.53209
               = 12.8 games
                          120 − 84                    14. f (x) = x3 + 4x2 − x − 1, x0 = −1
10. (a) L(x) = f (80) +            (x − 80)               f (x) = 3x2 + 8x − 1
                           80 − 60
152                                                    CHAPTER 3. APPLICATIONS OF DIFFERENTIATION

                     f (x0 )                                                                30
      (a) x1 = x0 −
                     f (x0 )
                      3       1
              = −1 −      =−                                                                20
                      −6      2
                     f (x1 )                                                            y
           x2 = x1 −
                     f (x1 )                                                                10

                  1   0.375
              =− −           = −0.4117647
                  2 −4.25
                                                                                                0
                                                                   −5.0         −2.5                0.0       2.5   5.0
      (b) The root is x ≈ −0.4064206546.                                            x
                                                                                            −10

                                                                 Start with x0 = −5 to find the root near −5:
 15. f (x) = x4 − 3x2 + 1 = 0, x0 = 1                            x1 = −4.718750, x2 = −4.686202,
     f (x) = 4x3 − 6x                                            x3 = −4.6857796, x4 = −4.6857795



                     f (x0 )
      (a) x1 = x0 −                                          18. From the graph, we see two roots:
                     f (x0 )
                      14 − 3 · 12 + 1              1
              =1−                              =                          15

                       4 · 13 − 6 · 1              2
                                                                          10
                      f (x1 )
           x2 = x1 −
                     f (x1 )                                               5

                                                                    -1          0           1             2     3   4
                        1 4          1 2                                   0
               1        2     −3     2     +1
              = −
               2              1 3          1                               -5
                         4    2     −6     2
                                                                          -10
                  5
              =
                  8                                                       -15


                                                                          -20

      (b) 0.61803



 16. f (x) = x4 − 3x2 + 1, x0 = −1                                                f (xi )
                                                                 Use xi+1 = xi −          with
     f (x) = 4x3 − 6x                                                             f (xi )
                                                                 f (x) = x4 − 4x3 + x2 − 1, and
                                                                 f (x) = 4x3 − 12x2 + 2x
                     f (x0 )                                     Start with x0 = 4 to find the root below 4:
      (a) x1 = x0 −                                              x1 = 3.791666667, x2 = 3.753630030, x3 =
                     f (x0 )
                   −1         1                                  3.7524339, x4 = 3.752432297
           = −1 −       =−                                       Start with x = −1 to find the root just above
                    2         2
                      f (x1 )                                    −1:
           x2 = x1 −                                             x1 = −0.7222222222,
                     f (x1 )
                                                                 x2 = −0.5810217936,
                1 0.3125                                         x3 = −0.5416512863,
           =− −               = −0.625
                2     2.5                                        x4 = −0.5387668233,
                                                                 x5 = −0.5387519962
      (b) The root is x ≈ −0.6180339887.



                       f (xi )                                                    f (xi )
 17. Use xi+1 = xi −           with                          19. Use xi+1 = xi −          with
                       f (xi )                                                    f (xi )
      f (x) = x3 + 4x2 − 3x + 1, and                             f (x) = x5 + 3x3 + x − 1, and
      f (x) = 3x2 + 8x − 3                                       f (x) = 5x4 + 9x2 + 1
3.1. LINEAR APPROXIMATIONS AND NEWTONS METHOD                                                                                             153

                                       10                                          x1 = −0.644108, x2 = −0.636751
                                                                                   x3 = −0.636733, x4 = −0.636733
                                                                                   Start with x0 = 1.5 to find the root near 1.5:
                                        5
                                                                                   x1 = 1.413799, x2 = 1.409634
                                                                                   x3 = 1.409624, x4 = 1.409624
                                        0

       −1.0         −0.5                     0.0           0.5           1.0
                                                                               22. Use xi+1 = xi − f (xii)) with
                                                                                                     f
                                                                                                       (x
                        x
                                                                                   f (x) = cos x2 − x, and
                                  y    −5                                          f (x) = 2x sin x2 − 1
                                                                                                                  3

                                      −10

                                                                                                                  2
    Start with x0 = 0.5 to find the root near 0.5:                                                            y


    x1 = 0.526316, x2 = 0.525262,                                                                                 1

    x3 = 0.525261, x4 = 0.525261
                                                                                                                  0
                      f (xi )                                                          -2        -1                   0       1       2

20. Use xi+1 = xi −           with                                                                                            x
                      f (xi )                                                                                    -1

    f (x) = cos x − x, and
    f (x) = − sin x − 1                                                                                          -2

                                      5.0

                                                                                   Start with x0 = 1 to find the root between 0
                                                                                   and 1:
                                      2.5
                                                                                   x1 = 0.8286590991, x2 = 0.8016918647,
                                                                                   x3 = 0.8010710854, x4 = 0.8010707652
                                      0.0
                                                                                                                  3
        −5    −4   −3       −2        −1     0     1   2         3   4   5
                        x
                                                                                                                  2
                                 y −2.5                                                                      y


                                                                                                                  1


                                  −5.0
                                                                                                                  0
    Start with x0 = 1 to find the root near 1:                                          -2        -1                   0       1
                                                                                                                              x
                                                                                                                                      2


    x1 = 0.750364, x2 = 0.739113,                                                                                -1

    x3 = 0.739085, x4 = 0.739085
                                                                                                                 -2



21. Use xi+1 = xi − f (xii)) with
                      f
                        (x
    f (x) = sin x − x2 + 1, and                                                                      f (xi )
    f (x) = cos x − 2x                                                         23. Use xi+1 = xi −           with
                                                                                                    f (xi )
                                      5.0
                                                                                   f (x) = ex + x, and
                                                                                   f (x) = ex + 1
                                                                                                            20
                                      2.5


                                                                                                            15
                                      0.0

        −5    −4   −3       −2        −1     0     1   2     3       4   5
                                                                                                           y 10
                        x

                                 y −2.5
                                                                                                             5



                                      −5.0
                                                                                                             0

                                                                                      −3    −2        −1              0   1       2   3
                                                                                                 x
    Start with x0 = −0.5 to find the root near                                                                −5

    −0.5:                                                                          Start with x0 = −0.5 to find the root between
154                                                 CHAPTER 3. APPLICATIONS OF DIFFERENTIATION

       0 and -1:                                              zeros of f ), Newton’s method will succeed.
       x1 = −0.566311, x2 = −0.567143                         Which root is found depends on the starting
       x3 = −0.567143, x4 = −0.567143                         place.
                      f (xi )                             33. f (x) = x2 + 1, x0 = 0
 24. Use xi+1 = xi −          with
                  √ f (xi )                                   f (x) = 2x
     f (x) = e−x − x, and                                                f (x0 )     1
                       1                                      x1 = x0 −          =0−
     f (x) = −e−x − √                                                    f (x0 )     0
                     2 x                                      The method fails because f (x0 ) = 0. There
                                                              are no roots.
          1
                                                          34. Newton’s method fails because the function
        0.5
                                                              does not have a root!
                                                                     4x2 − 8x + 1
          0
               0   0.5    1       1.5     2
                                                          35. f (x) =              = 0, x0 = −1
                                                                     4x2 − 3x − 7
                                                              Note: f (x0 ) = f (−1) is undefined, so New-
        -0.5
                                                              ton’s Method fails because x0 is not in the do-
                                                              main of f . Notice that f (x) = 0 only when
         -1
                                                              4x2 − 8x + 1 = 0. So using Newton’s Method
                                                              on g(x) = 4x2 − 8x + 1 with x0 = −1 leads to
                                                              x ≈ .1339. The other root is x ≈ 1.8660.
       Start with x0 = 0.5 to find the root close to
                                                          36. The slope tends to infinity at the zero. For ev-
       0.5:
                                                              ery starting point, the sequence does not con-
       x1 = 0.4234369253, x2 = 0.4262982542,
                                                              verge.
       x3 = 0.4263027510
                                √                         37. (a) With x0 = 1.2,
 25.   f (x) = x2 − 11; x0 = 3; 11 ≈ 3.316625
                             √                                    x1 = 0.800000000,
 26.   Newton’s method for x near x = 23 is xn+1 =                x2 = 0.950000000,
       1
       2 (xn + 23/xn ). Start with x0 = 5 to get:
                                                                  x3 = 0.995652174,
       x1 = 4.8, x2 = 4.7958333, and x3 = 4.7958315.              x4 = 0.999962680,
                                √                                 x5 = 0.999999997,
 27.   f (x) = x3 − 11; x0 = 2; 3 11 ≈ 2.22398                    x6 = 1.000000000,
                                √                                 x7 = 1.000000000
 28.   Newton’s method for 3 x near x = 23 is
       xn+1 = 1 (2xn + 23/x2 ). Start with x0 = 3
                3             n                                (b) With x0 = 2.2,
       to get:                                                     x0 = 2.200000, x1 = 2.107692,
       x1 = 2.851851851, x2 = 2.843889316, and                     x2 = 2.056342, x3 = 2.028903,
       x3 = 2.884386698                                            x4 = 2.014652, x5 = 2.007378,
                                   √                               x6 = 2.003703, x7 = 2.001855,
 29.   f (x) = x4.4 − 24; x0 = 2; 4.4 24 ≈ 2.059133
                                                                   x8 = 2.000928, x9 = 2.000464,
                                 √
 30.   Newton’s method for 4.6 x near x = 24 is                    x10 = 2.000232, x11 = 2.000116,
                1
       xn+1 = 4.6 (3.6xn +24/x3.6 ). Start with x0 = 2
                                n                                  x12 = 2.000058, x13 = 2.000029,
       to get:                                                     x14 = 2.000015, x15 = 2.000007,
       x1 = 1.995417100, x2 = 1.995473305, and                     x16 = 2.000004, x17 = 2.000002,
       x3 = 1.995473304                                            x18 = 2.000001, x19 = 2.000000,
                                                                   x20 = 2.000000
 31. f (x) = 4x3 − 7x2 + 1 = 0, x0 = 0                             The convergence is much faster with x0 =
     f (x) = 12x2 − 14x                                            1.2.
                f (x0 )       1
     x1 = x0 −          =0−
                f (x0 )       0                           38. Starting with x0 = 0.2 we get a sequence that
     The method fails because f (x0 ) = 0. Roots              converges to 0 very slowly. (The 20th itera-
     are 0.3454, 0.4362, 1.659.                               tion is still not accurate past 7 decimal places).
                                                              Starting with x0 = 3 we get a sequence that
 32. Newton’s method fails because f = 0. As long
                                           7                  quickly converges to π. (The third iteration is
     as the sequence avoids xn = 0 and xn = (the              already accurate to 10 decimal places!)
                                           6
3.1. LINEAR APPROXIMATIONS AND NEWTONS METHOD                                                                155

                                                                    √
39. (a) With x0 = −1.1                                  43. f (x) = √ 4 + x
        x1 = −1.0507937,                                    f (0) = 4 + 0 = 2
        x2 = −1.0256065,                                             1
                                                            f (x) = (4 + x)−1/2
        x3 = −1.0128572,                                             2
                                                                     1              1
        x4 = −1.0064423,                                    f (0) = (4 + 0)−1/2 =
        x5 = −1.0032246,                                             2              4
                                                                                             1
        x6 = −1.0016132,                                    L(x) = f (0) + f (0)(x − 0) = 2 + x
                                                                                             4
        x7 = −1.0008068,                                                    1
        x8 = −1.0004035,                                    L(0.01) = 2 + (0.01) = 2.0025
                                                                       √ 4
        x9 = −1.0002017,                                    f (0.01) = 4 + 0.01 ≈ 2.002498
        x10 = −1.0001009,                                                 1
                                                            L(0.1) = 2 + (0.1) = 2.025
        x11 = −1.0000504,                                             √ 4
        x12 = −1.0000252,                                   f (0.1) = 4 + 0.1 ≈ 2.0248
        x13 = −1.0000126,                                               1
                                                            L(1) = 2 + (1) = 2.25
        x14 = −1.0000063,                                           √   4
        x15 = −1.0000032,                                   f (1) = 4 + 1 ≈ 2.2361
        x16 = −1.0000016,
        x17 = −1.0000008,
        x18 = −1.0000004,
        x19 = −1.0000002,
        x20 = −1.0000001,                               44. The linear approximation for ex at x = 0 is
        x21 = −1.0000000,                                   L(x) = 1 + x. The error when x = 0.01 is
        x22 = −1.0000000                                    0.0000502, when x = 0.1 is 0.00517, and when
     (b) With x0 = 2.1                                      x = 1 is 0.718.
         x0 = 2.100000000,
         x1 = 2.006060606,
         x2 = 2.000024340,
         x3 = 2.000000000,
         x4 = 2.000000000                               45. (a) f (0) = g(0) = h(0) = 1, so all pass
         The rate of convergence in (a) is slower               through the point (0, 1).
         than the rate of convergence in (b).                   f (0) = 2(0 + 1) = 2,
                                                                g (0) = 2 cos(2 · 0) = 2, and
40. From exercise 37, f (x) = (x − 1)(x − 2)2 . New-
                                                                h (0) = 2e2·0 = 2,
    ton’s method converges slowly near the double
                                                                so all have slope 2 at x = 0.
    root. From exercise 39, f (x) = (x − 2)(x + 1)2 .
                                                                The linear approximation at x = 0 for all
    Newton’s method again converges slowly near
                                                                three functions is L(x) = 1 + 2x.
    the double root. In exercise 38, Newton’s
    method converges slowly near 0, which is a zero
    of both x and sin x but converges quickly near
    π, which is a zero only of sin x.                        (b) Graph of f (x) = (x + 1)2 :
                                                                                        5
41. f (x) = tan x, f (0) = tan 0 = 0
    f (x) = sec2 x, f (0) = sec2 0 = 1                                                  4

    L(x) = f (0) + f (0)(x − 0) L(0.01) = 0.01
                                                                                        3
           = 0 + 1(x − 0) = x                                                       y

    f (0.01) = tan 0.01 ≈ 0.0100003                                                     2

    L(0.1) = 0.1                                                                        1
    f (0.1) = tan(0.1) ≈ 0.1003
    L(1) = 1                                                                            0

    f (1) = tan 1 ≈ 1.557                                            −3   −2       −1        0   1   2   3
                                                                                        −1
                                  √                                            x
42. The linear approximation for 1 + x at x = 0
                   1
    is L(x) = 1 + 2 x. The error when x = 0.01 is
    0.0000124, when x = 0.1 is 0.00119, and when
    x = 1 is 0.0858.                                             Graph of f (x) = 1 + sin(2x):
156                                                                   CHAPTER 3. APPLICATIONS OF DIFFERENTIATION

                                     5
                                                                                                           2

                                     4

                                                                                                           1
                                     3
                             y
                                     2                                                                     0
                                                                                           -2      -1           0    1        2
                                                                                                                     x
                                     1
                                                                                                           -1

                                     0
             −3   −2        −1                0   1       2   3
                                                                                                           -2
                       x             −1



                                                                                       Graph of h(x) = sinh x:

          Graph of f (x) = e2x :
                                                                                                           3
                                     5
                                                                                                           2

                                     4
                                                                                                           1


                                     3                                                                     0
                                                                                           -2      -1           0    1        2
                                 y
                                                                                                           -1        x
                                     2

                                                                                                           -2

                                     1
                                                                                                           -3


                                     0

            −3    −2        −1                0   1       2       3
                       x             −1
                                                                                       sin x is the closest fit, but sinh x is close.

                                                                                       √
                                                                                       4
                                                                            47. (a)      16.04 = 2.0012488
                                                                                       L(0.04) = 2.00125
                                                                                       |2.0012488 − 2.00125| = .00000117
 46. (a) f (0) = g(0) = h(0) = 0, so all pass                                          √
                                                                                       4
         through the point (0, 0).                                               (b)     16.08 = 2.0024953
         f (0) = cos 0 = 1,                                                            L(.08) = 2.0025
                    1                                                                  |2.0024953 − 2.0025| = .00000467
         g (0) =         = 1, and
                  1 + 02                                                               √
         h (0) = cosh 0 = 1,                                                     (c)   4
                                                                                         16.16 = 2.0049814
         so all have slope 1 at x = 0.                                                 L(.16) = 2.005
         The linear approximation at x = 0 for all                                     |2.0049814 − 2.005| = .0000186
         three functions is L(x) = x.


      (b) Graph of f (x) = sin x:                                           48. If you graph | tan x − x|, you see that the dif-
                                                                                ference is less than .01 on the interval −.306 <
                                          2                                     x < .306 (In fact, a slightly larger interval
                                                                                would work as well).
                                          1



                                                                            49. The first tangent line intersects the x-axis at a
                                          0
             -2        -1                     0       1       2                 point a little to the right of 1. So x1 is about
                                                      x
                                                                                1.25 (very roughly). The second tangent line
                                         -1
                                                                                intersects the x-axis at a point between 1 and
                                                                                x1 , so x2 is about 1.1 (very roughly). Newton’s
                                         -2                                     Method will converge to the zero at x = 1.
                                                                                Starting with x0 = −2, Newton’s method con-
          Graph of g(x) = tan−1 x:                                              verges to x = −1.
3.1. LINEAR APPROXIMATIONS AND NEWTONS METHOD                                                     157

                                                            f (x) = 2x − 1
                          3
                                                                      3
                                                            At x0 =
                          2
                                                                      2
                                                                            2
                      y                                                 3        3      1
                                                            f (x0 ) =         − −1=−
                          1                                             2        2      4
                                                            and
                                                                           3
        -2      -1
                          0
                               0   1     2
                                                            f (x0 ) = 2         −1=2
                                   x
                                                                           2
                          -1                                By Newton’s formula,
                                                                         f (x0 )   3 −1   13
                                                            x1 = x0 −            = − 4 =
                          -2                                            f (x0 )    2  2   8

    Starting with x0 = 0.4, Newton’s method con-        (b) f (x) = x2 − x − 1
    verges to x = 1.                                        f (x) = 2x − 1
                                                                      5
                                                            At x0 = 3
                          3                                                 2
                                                                        5        5    1
                                                            f (x0 ) =         − −1=
                          2                                             3        3    9
                      y
                                                            and
                                                                           5        7
                          1                                 f (x0 ) = 2         −1=
                                                                           3        3
                          0
                                                            By Newton’s formula,
        -2      -1             0   1     2
                                                                         f (x0 )
                                   x                        x1 = x0 −
                          -1                                            f (x0 )
                                                                         1
                                                                   5     9       5   1   34
                          -2                                   =     −   7   =     −   =
                                                                   3     3
                                                                                 3 21    21

50. It wouldn’t work because f (0) = 0. x0 = 0.2        (c) f (x) = x2 − x − 1
    works better as an initial guess. After jumping         f (x) = 2x − 1
                                                                      8
    to x1 = 2.55, the sequence rapidly decreases            At x0 = 5
                                                                             2
    toward x = 1. Starting with x0 = 10, it takes                       8        8       1
                                                            f (x0 ) =          − −1=−
    several steps to get to 2.5, on the way to x = 1.                   5        5       25
                                                            and
                   f (xn )                                                 8        11
51. xn+1 = xn −                                             f (x0 ) = 2         −1=
                   f (xn )                                                 5         5
                     x2 − c
                       n                                    By Newton’s formula,
           = xn −                                                        f (x0 )
                       2xn                                  x1 = x0 −
                    x2       c                                          f (x0 )
           = xn − n +                                              8 − 25  1
                                                                                 8  1    89
                   2xn      2xn                                 = − 11 = +             =
             xn      c                                             5             5 55    55
           =    +                                                         5
              2     2xn
             1           c                              (d) From part (a),
           =    xn +                                                   F4              F7
             2          xn                                  sincex0 =     , hence x1 =    .
             √                   √          √                          F3              F6
    If x0 < a, then a/x0 > a, so x0 < a <                   From part (b),
    a/x0 .                                                              F5             F9
                             √                              since x0 =      hence x1 =    .
52. The root of xn − c is n c, so Newton’s method                       F4             F8
                                                            From part (c),
    approximates this number.                                           F6             F11
    Newton’s method gives                                   since x0 =      hence x1 =      .
                  f (xi )        xn − c                                 F5             F10
    xi+1 = xi −            = xi − i n−1                                              Fn+1
                  f (xi )        nxi                        Thus in general if x0 =       , then x1 =
                                                                                      Fn
             1                                              F2n+1
           = (nxi − xi + cx1−n ),
                               i                                   implies m = 2n + 1 and k = 2n
             n                                                F2n
    as desired.
                                                                             3            Fn+1
53. (a) f (x) = x2 − x − 1                              (e) Given x0 =         , then lim      will be
                                                                             2        n→∞ Fn
158                                                   CHAPTER 3. APPLICATIONS OF DIFFERENTIATION

           the zero of the function f (x) = x2 −                                                 2P x
                                                                 L(x) = 120 − .01(120) = P −
           x − 1 which is 1.618034. Therefore,                                                    R
                Fn+1                                                           2 · 120x
            lim       = 1.618034                                       = 120 −
           n→∞ Fn                                                                  R
                                                                        2x
                                                                 .01 =
 54. The general form of functionf (x) is,                              R
              1 n+2               1           1                   x = .005R = .005(20,900,000)
     fn (x) =    2   x − 3 for n < x < n−1 .
              5                  2          2                      = 104,500 ft
     Hence
                      2n+2       1           1              58. If m = m0 (1 − v 2 /c2 )1/2 , then
     f (x) = fn (x) =       for n < x < n−1 .
                        5       2          2                    m = (m0 /2)(1 − v 2 /c2 )−1/2 (−2v/c2 ), and
     By Newton’s method,
                                                                m = 0 when v = 0. The linear approxima-
            3   f 34     3    f1 34
     x1 = −           = −                                       tion is the constant function m = m0 for small
            4 f 3  4
                         4 f1 4    3
                                                                values v.
            3 (3/5 )      3    x0
           =  −          = =                                59. The only positive solution is 0.6407.
            4 (8/5 )      8     2
                       x1   x0            x0
      Similarly, x2 =     = 2 and x3 = 3                    60. The smallest positive solution of the first equa-
                        2   2              2                    tion is 0.132782, and for the second equa-
                                          x0
      Continuing this, we get, xn−1 = n−1 It may                tion the smallest positive solution is 1, so the
                                        2
      also be observed that, for each fn (x)                    species modeled by the second equation is cer-
            (1/2n ) + 1/2n+1        3                           tain to go extinct. This is consistent with the
      x0 =                     = n+1 ,
                     2            2                             models, since the expected number of offspring
             x0        3                   3                    for the population modeled by the first equa-
      xn = n = 2n+1 ⇒ xn+1 = 2n+2 which
             2       2                  2                       tion is 2.2, while for the second equation it is
      is the zero of F . Therefore Newton’s method
                                                                only 1.3
      converges to zero of F .
                                                            61. The linear approximation for the inverse tan-
 55. For small x we approximate ex by x + 1                     gent function at x = 0 is
     (see exercise 44)                                          f (x) ≈ f (0) + f (0)(x − 0)
      Le2πd/L − e−2πd/L                                         tan−1 (x) ≈ tan−1 (0) + 1+02 (x − 0)
                                                                                           1

       e2πd/L + e−2πd/L                                         tan−1 (x) ≈ x
         L 1 + 2πd − 1 − 2πd
                   L         L
                                                                Using this approximation,
      ≈                                                                       3[1 − d/D] − w/2
            1 + 2πd + 1 − 2πd
                 L          L                                    φ = tan−1
                4πd
                                                                                    D−d
           L     L
       ≈            = 2πd                                             3[1 − d/D] − w/2
              2                                                  φ≈
               4.9                                                          D−d
      f (d) ≈      · 2πd = 9.8d                                 If d = 0, then φ ≈ 3−w/2 . Thus, if w or D
                π                                                                        D
                                                                increase, then φ decreases.
                    8πhcx−5
 56. If f (x) =                  , then using the linear    62. d (θ) = D(w/6 sin θ)
                  ehc/(kT x) − 1                                d(0) = D(1 − w/6) so
      approximation we see that
                 8πhcx−5                                        d(θ) ≈ d(0) + d (0)(θ − 0)
      f (x) ≈        hc
                               = 8πkT x−4                             = D(1 − w/6) + 0(θ) = D(1 − w/6),
              (1 + kT x ) − 1
      as desired.                                                as desired.

                P R2
 57. W (x) =
              (R + x)2
                       , x0 = 0                            3.2     Indeterminate Forms and
      W (x) =
                −2P R2                                             L’Hˆpital’s Rule
                                                                      o
               (R + x)3
      L(x) = W (x0 ) + W (x0 )(x − x0 )                                 x+2
                                                             1. lim
                                                                 x→−2   x2 − 4
                  P R2        −2P R2                                       x+2
               =         +               (x − 0)                 = lim
                (R + 0)2     (R + 0)3                             x→−2 (x + 2)(x − 2)
                    2P x                                                 1        1
               =P−                                               = lim       =−
                     R                                            x→−2 x − 2      4
ˆ
3.2. INDETERMINATE FORMS AND L’HOPITAL’S RULE                                                     159

          x2 − 4                                           sin x − x              0
 2. lim                                            11. lim       3
                                                                         is type ;
    x→2 x2− 3x + 2                                     x→0     x                  0
          (x − 2)(x + 2)                               we apply L’Hˆpital’s Rule thrice to get
                                                                       o
    = lim                                                     cos x − 1           − sin x
      x→2 (x − 2)(x − 1)                               = lim                = lim
          x+2                                            x→0       3x2        x→0   6x
    = lim       =4                                            − cos x         1
      x→2 x − 1                                        = lim             =− .
                                                         x→0       6          6
        3x2 + 2
 3. lim                                                    tan x − x         0
    x→∞ x2 − 4                                     12. lim            is type ;
               2
          3 + x2
                                                       x→0    x3             0
    = lim                                              we apply L’Hˆpital’s Rule to get
                                                                    o
      x→∞ 1 − 4                                            sec2 x − 1
              x2
      3                                                lim            .
    = =3                                               x→0    3x2
      1
                                                       Apply L’Hˆpital’s Rule twice more to get
                                                                  o
           x+1              ∞                              2 sec2 x tan x
 4. lim             is type   ;                        lim
   x→−∞ x2 + 4x + 3         ∞                          x→0       6x
                                                              4 sec2 x tan2 x + 2 sec4 x  1
    we apply L’Hˆpital’s Rule to get
                o                                      = lim                             = .
             1                                           x→0               6              3
     lim         = 0.                                      √               √          √
    x→−∞ 2x + 4
                                                             t−1             t−1        t+1
                                                   13. lim          = lim          · √
        e2t − 1            0                           t→1 t − 1       t→1 t − 1        t+1
 5. lim            is type ;
    t→0    t               0                                                   (t − 1)
    we apply L’Hˆpital’s Rule to get
                     o                                              = lim          √
                                                                       t→1 (t − 1) t + 1
         d
             e2t − 1
    lim dt d                                                                  1        1
                                                                    = lim √         =
              dt t
    t→0
                                                                       t→1   t+1       2
        2e2t       2
    lim        = =2                                            ln t         0
    t→0 1          1                               14. lim          is type ;
         sin t            0
                                                       t→1 t    −1          0
 6. lim          is type ;
    t→0 e3t−1             0                            we apply L’Hˆpital’s Rule to get
                                                                      o
    we apply L’Hˆpital’s Rule to get
                   o                                        d                1
                                                            dt (ln t)
          d
            (sin t)         cos t 1                    lim d           = lim t = 1
                                                                         t→1 1
                                                           dt (t − 1)
                                                       t→1
    lim ddt 3t        = lim 3t =
                        t→0 3e    3
        dt (e − 1)
    t→0
                                                            x3        ∞
        tan−1 t             0                      15. lim x is type     ;
 7. lim            is type ;                           x→∞ e          ∞
    t→0 sin t               0                          we apply L’Hˆpital’s Rule thrice to get
                                                                   o
    we apply L’Hˆpital’s Rule to get
                     o
         d
             tan−1 t          1/(1 + t2 )                  3x2       6x
    lim dt d            = lim             =1            lim    = lim x
    t→0
           dt (sin t)
                          t→0    cos t                 x→∞ ex   x→∞ e
                                                                     6
          sin t             0                                  = lim x = 0.
 8. lim           is type     ;                                 x→∞ e
    t→0 sin−1 t             0
                                                           ex         ∞
    we apply L’Hˆpital’s Rule to get
                   o                               16. lim    is type   ;
          d                                            x→∞ x4         ∞
         dt (sin t)           cos t
    lim              = lim    √         =1             we apply L’Hˆpital’s Rule four times to get
                                                                   o
    t→0 d sin−1 t      t→0 1/( 1 − t2 )
        dt                                                  ex           ex
                                                        lim   3
                                                                = lim
          sin 2x         0                             x→∞ 4x     x→∞ 12x2
 9. lim          is type ;                                              ex          ex
    x→π    sin x         0                                      = lim        = lim      = ∞.
                                                                  x→∞ 24x      x→∞ 24
    we apply L’Hˆpital’s Rule to get
                o
                                                               x cos x − sin x         ∞
        2 cos 2x   2(1)                            17. limx→0           2      is type   ;
    lim          =      = −2.                                      x sin x             ∞
    x→π cos x      −1                                  we apply L’Hˆpital’s Rule twice to get
                                                                      o
                                                               cos x − x sin x − cos x
          cos−1 x                                      limx→0
10. lim           is undefined (numerator goes to               sin2 x + 2x sin x cos x
    x→−1 x2 − 1                                                       −x sin x
    π, denominator goes to 0).                         = lim
                                                          x→0 sin x (sin x + 2x cos x)
160                                                     CHAPTER 3. APPLICATIONS OF DIFFERENTIATION

                   −x                                                                x − π cos x
      = lim                                                       = lim                  2
                                                                                                      = 0
        x→0 sin x + 2x cos x
                       −1
                                                                       π
                                                                    x→ 2        cos x − x − π sin x
                                                                                            2
      = lim
        x→0 cos x + 2 cos x − 2x sin x                                 ln x        ∞
          1                                                   21. lim 2    is type
      =− .                                                        x→∞ x            ∞
          3                                                       we apply L’Hˆpital’s Rule to get
                                                                               o
                                                                       1/x         1
 18. Rewrite as one fraction, we have                              lim     = lim        = 0.
                                                                  x→∞ 2x      x→∞ 2x2
                   1                  x cos x − sin x
     lim cot x −          = lim                                        ln x        ∞
     x→0           x           x→0        x sin x             22. lim √    is type    ;
                       0                                          x→∞     x        ∞
     which is of type                                             we apply L’Hˆpital’s Rule to get
                                                                               o
                       0                                                1
     we apply L’Hˆpital’s Rule to get
                   o                                                                2
                cos x − x sin x − cos x                            lim x = lim √ = 0.
                                                                        1
      = lim
                                                                  x→∞ √
                                                                       2 x
                                                                              x→∞    x
        x→0         sin x + x cos x
                                                                       t          ∞
                         d
                             (−x sin x)                       23. lim t is type
       = lim            dx                                        t→∞ e           ∞
         x→0       d
                       (sin x + x cos x)                          we apply L’Hˆpital’s Rule to get
                                                                                o
                  dx                                                     d
                                                                           (t)       1
                     − sin x − x cos x                             lim dt      = lim t = 0.
       = lim                                =0                    t→∞ d (et )    t→∞ e
         x→0      cos x + cos x − x sin x                              dt

                                                                        sin 1
                                                                            t       0
 19. Rewrite as one fraction, we have                         24. lim      1    is type
                                                                  t→∞
                                                                           t
                                                                                    0
           x+1            2                                       we apply L’Hˆpital’s Rule to get
                                                                               o
     lim            −
     x→0      x        sin 2x                                             - 1 cos 1
                                                                            2                 1
              (x + 1) sin 2x − 2x             0                   = lim t 1 t = lim cos = 1.
     = lim                             is type ;                     t→∞    − t2      t→∞      t
       x→0            x sin 2x                0
     we apply L’Hˆpital’s Rule four times to get
                      o                                               ln (ln t)
            d                                                 25. lim
               (x + 1) sin 2x − 2x                                t→1   ln t
     lim dx d                                                     As t approaches ln from below, ln t is a small
                  dx (x sin 2x)
     x→0
                                                                  negative number. Hence ln (ln t) is undefined,
              sin 2x + 2(x + 1) cos 2x − 2                        so the limit is undefined.
     = lim
       x→0           sin 2x + 2x cos 2x
                d
                                                                         sin (sin t)           0
                   (sin 2x + 2(x + 1) cos 2x − 2)             26. lim                  is type
     = lim dx d                                                   t→0       sin t              0
                      dx (sin 2x + 2x cos 2x)
       x→0                                                        we apply L’Hˆpital’s Rule to get
                                                                                  o
              2 cos 2x + 2 cos 2x − 4(x + 1) sin 2x                      cos (sin t) cos t
     = lim                                                        lim                       = 1.
       x→0        2 cos 2x + 2 cos 2x − 4x sin 2x                 t→0           cos t
        4
     = =1                                                               sin (sinh x)           0
        4                                                     27. lim                  is type
                                                                  x→0   sinh (sin x)           0
                                                                  we apply L’Hˆpital’s Rule to get
                                                                                o
                         1                                              cos (sinh x) cosh x
 20. lim      tan x +                                             lim                         =1
        π
      x→ 2            x− π  2
                                                                  x→0    cosh (sin x) cos x
      In this case the limit has the form (∞ - ∞).                      sin x − sinh x
                           sin x                              28. lim
      Rewrite tan x as            and then as one frac-           x→0   cos x − cosh x
                           cos x
      tion, we get                                                          2 sin x − ex + e−x
                                                                  = lim
                         1                                          x→0    2 cos x − ex − e−x
       lim tan x +
      x→ 2π
                      x− π  2
                                                                            2ex sin x − e2x + 1            0
                                                                  = lim                            is type
                sin x        1                                      x→0    2ex cos x − e2x − 1             0
      = lim            +                                          we apply L’Hˆpital’s Rule twice to get
                                                                                o
         x→ π
            2  cos x x − π     2
                                                                          2ex cos x + 2ex sin x − 2e2x
                 x − π sin x + cos x
                       2                         0                lim
      = lim                              is type                  x→0    −2ex sin x + 2ex cos x − 2e2x
         x→ π
            2         x − π cos x
                           2
                                                 0
                                                                           cos x + sin x − 1             0
      we apply L’Hˆpital’s Rule to get
                     o                                            = lim                         is type
                                                                    x→0    cos x − sin x − 1             0
                sin x + x − π cos x − sin x
                                2                                          − sin x + cos x
      = lim                                                       = lim                       = −1
         x→ π
            2       cos x − x − π sin x
                                    2                               x→0    − sin x − cos x
ˆ
3.2. INDETERMINATE FORMS AND L’HOPITAL’S RULE                                                                                  161

           ln x          ∞                                                                         x+1
                                                                                                      
29. lim          is type
    x→0 + cot x          ∞                                                           ln           x−2 
    we apply L’Hˆpital’s Rule to get
                    o                                                         = lim                   
                                                                               x→∞               √ 1  
             1/x                                                                                       x2 −4
     lim
    x→0+ − csc2 x                                                                                             0
                        sin x                                       This last limit has indeterminate form , so
    = lim+ − sin x ·          = (0)(1) = 0.                                                                   0
       x→0                x                                         we can apply L’Hˆpital’s Rule and simplify to
                                                                                     o
                                                                    find that the above is equal to
             √
           x                                                              −3(x2 − 4)3/2
30. lim+     = 0 (numerator goes to 0 and de-                        lim                 and this is equal to 3. So
    x→0 ln x                                                        x→∞ −x3 + x2 + 2x
    nominator goes to −∞).                                           lim ln y = 3.
                                                                    x→∞
                                                                    Thus lim y = lim eln y = e3 ≈ 20.086.
                                                                           x→∞           x→∞
31. lim          x2 + 1 − x                                                          √
    x→∞
                           √                                                 1          x
                             x2 + 1 + x                         35. lim+    √ −√
    = lim       x 2+1−x √                                           x→0       x√     x+1    √
     x→∞                     x2 + 1 + x                                          x + 1 − ( x)2
              2
            x +1−x     2                                            = lim+        √ √
    = lim √                                                           x→0
                                                                               √ x x+1
     x→∞      x2 + 1 + x                                                         x+1−x
                1                                                   = lim       √ √
    = lim √             =0                                            x→0+       x x+1
     x→∞    x 2+1+x                                                 = ∞.
                                                                         √
                                                                            5−x−2               0
                                  ln x
                                         −1                     36. lim √               is type
32. lim ln x − x = lim             x
                                               = −∞ since the       x→1    10 − x − 3           0
                                         1
    x→∞                    x→∞
                                         x
                                                                    we apply L’Hˆpital’s Rule to get
                                                                                   o
    numerator goes to −1 and the denominator                             1         −1/2
                                                                         2 (5 − x)      (−1)
    goes to 0+ . (Recall Example 2.8 which shows                    lim 1
                                                                    x→1 (10 − x)−1/2 (−1)
                                                                        2 √
         ln x
     lim      = 0.)                                                           10 − x     3
    x→∞ x                                                           = lim √          = .
                                                                      x→1      5−x       2
                             x                                                           x
                       1                                        37. Let y = (1/x) . Then ln y = x ln(1/x). Then
33. Let y =       1+
                       x                                             lim+ ln y = lim+ x ln(1/x) = 0, by Exercise
                                                                    x→0              x→0
                       1
    ⇒ ln y = x ln 1 +       . Then                                  27. Thus lim+ y = lim+ eln y = 1.
                       x                                                       x→0               x→0
                                 1
     lim ln y = lim x ln 1 +                                    38. Let y = lim+ (cos x)1/x . Then
    x→∞        x→∞               x                                            x→0
                    ln 1 + x  1                                                  1
              = lim                                                 ln y = lim+     ln cos x
                                                                          x→0 x
               x→∞      1/x
                      1        1                                                 ln(cos x)           0
                    1+ x1   − x2                                        = lim                is type
              = lim                                                       x→0  +      x              0
               x→∞     −1/x2                                        so apply L’Hˆpital’s Rule to get
                                                                                  o
                       1                                                  − tan x
              = lim          = 1.                                    lim+          = 0.
               x→∞ 1 + 1                                            x→0      1
                          x
    Hence lim y = lim eln y = e.                                    Therefore the limit is y = e0 = 1.
             x→∞           x→∞
                                                                                     t                         t
                                                                            t−3                    (t − 3)
                                                                39. lim                  = lim
34. Notice that the limit in question has the inde-                 t→∞     t+2              t→∞ (t + 2)t
    terminate form 1∞ . Also, note that as x gets                                    3 t        lim 1 − 3
                                                                                                                       t
           x+1       x+1                                                      1−     t         t→∞         t
    large,        =        .                                        = lim                    =
                                                                        t→∞          2 t                   2           t
           x−2       x −√2                                                    1+     t          lim 1 + t
                                                                                                 t→∞
                                  x2 −4                                              −3 t
                      x+1                                               lim 1 +
    Define y =                                . Then                     t→∞           t           e−3
                      x−2                                           =                         =       = e−5
                                                                                     2 t           e2
             √                   x+1                                      lim 1 +    t
                                                                        t→∞
    ln y =       x2 − 4 ln                   and
                                 x−2                                                     t                                 t
                                                                                                                   3
                                                   x+1                      t−3                          1−        t
     lim ln y = lim              x2 − 4 ln                      40. lim                      = lim                 1
    x→∞             x→∞                            x−2              t→∞     2t + 1            t→∞        2+        t
162                                                  CHAPTER 3. APPLICATIONS OF DIFFERENTIATION

                       3 t
                1−     t                e−3                          we apply L’Hˆpital’s Rule to get
                                                                                 o
      = lim                  t   = lim         =0
        t→∞            1/2         t→∞ 2t e1/2
              2t 1 +                                                     n cos nx  n
                        t                                            lim          = .
                                                                     x→0 m cos mx  m
 41. L’Hˆpital’s rule does not apply. As x → 0, the
         o                                                              sin x2       2x cos x2
     numerator gets close to 1 and the denominator         50. (a) lim     2
                                                                               = lim
                                                                     x→0 x       x→0    2x
     is small and positive. Hence the limit is ∞.
                                                                               = lim cos x2 = 1,
          ex − 1         0           ex                                           x→0
 42. lim         is type , but lim       is not, so                                           sin x
     x→0    x 2          0      x→0 2x                             which is the same as lim         .
     L’Hˆpital’s Rule does not apply to this limit.
         o                                                                                x→0 x

                                                                        1 − cos x2
 43. L’Hˆpital’s rule does not apply. As x → 0, the
         o                                                     (b) lim
                                                                   x→0      x4
     numerator is small and positive while the de-                        2x sin x2        sin x2
     nominator goes to −∞. Hence the limit is 0.                   = lim        3
                                                                                     = lim
                                                                     x→0     4x        x→0 2x2
                2x                                                    1      sin x 2
                                                                                      1
     Also lim      , which equals lim x2 , is not of               = lim             = (by part (a)),
           x→0 2/x                 x→0
                                                                      2 x→0 x2        2
               0
     the form     so L’Hˆpital’s rule doesn’t apply
                         o
               0                                                     while
     here either.
           sin x        0           cos x                                1 − cos x        sin x    1       1
 44. lim         is type , but lim        is not, so                 lim      2
                                                                                   = lim        = (1) =
      x→0 x2            0       x→0 2x                               x→0    x         x→0 2x       2       2
      L’Hˆpital’s rule does not apply. This limit is
          o                                                          so both of these limits are the same.
      undefined because the numerator goes to 1 and             (c) Based on the patterns found in exercise
      the denominator goes to 0.                                   45, we should guess
           csc x                                                       sin x3              1 − cos x3  1
 45. lim+ √                                                        lim        = 1 and lim             = .
     x→0      x                                                    x→0 x3              x→0     x6      2
                                     ∞
     In this case limit has the form   , L’Hˆspital’s
                                            o
                                     0                             (x + 1)(2 + sin x)
     Rule should not be used.                              51. (a)
                                                                      x(2 + cos x)
            x−3/2             ∞                                     x
 46. lim+          is type       .    In this case             (b) x
      x→0    ln x            −∞                                    e
      L’Hˆspital’s Rule should be used.
          o                                                        3x + 1
                                                               (c)
            x2 − 3x + 1                                             x−7
 47. lim                = ∞. In this case limit has                3 − 8x
      x→∞     tan−1 x                                          (d)
      the form ∞. So L’Hˆspital’s Rule should not
                          o                                        1 + 2x
      be used.
                                                           52. (a) lim x − ln x = ∞ (see exercise 32).
          ln x2           ∞                                        x→∞
 48. lim          is type   . So L’Hˆspital’s Rule
                                    o                                  √
     x→∞ ex/3             ∞                                    (b) lim x2 + 1 − x = 0 (see exercise 31).
     should be used.                                               x→∞
                                                                       √
                                   sin 3x                      (c) lim x2 + 4x − x
 49. (a) Starting with            lim     , we cannot              x→∞     √
                                   sin 2x
                                 x→0                               = lim ( x2 + 4x − x)
                                    3x                               x→∞
          “cancel sin”to get lim       . We can cancel                          4x
                               x→0 2x                              = lim √
          the x’s in the last limit to get the final an-              x→∞    x2 + 4x + x
          swser of 3/2. The first step is likely to give                              1
                                                                                 4x x
          a correct answer because the linear ap-                  = lim √
                                                                     x→∞                  1
          proximation of sin 3x is 3x, and the linear                     ( x2 + 4x + x)
                                                                                         x
          approximation of sin 2x is 2x. The linear                            4
                                                                   = lim               = 2,
          approximations are better the closer x is                  x→∞         4
                                                                             1+ x +1
          to zero, so the limits are likely to be the
                                                                   where to get from the second to
          same.
                                                                   the third line, we have multiplied by
                                                                    √
               sin nx
      (b) lim          is type 0 ;
                                0
                                                                   ( x2 + 4x + x)
          x→0 sin mx                                                √              .
                                                                   ( x2 + 4x + x)
ˆ
3.2. INDETERMINATE FORMS AND L’HOPITAL’S RULE                                                                                 163

53. lim ex = lim xn = ∞                                        In general,when the degree of exponential term
    x→∞        x→∞
          ex                                                   in the numerator and denominator are differ-
     lim n = ∞.           Since n applications of                                   ln ekx + p(x)
    x→∞ x                                                      ent, then the lim                    for polyno-
    L’Hˆpital’s rule yields
        o                                                                    x→∞ ln (ecx + q(x))
          ex                                                   mials p and q and positive numbers. k and c
     lim      = ∞.
    x→∞ n!                                                     will be the fraction of degrees that is k .
                                                                                                       c
    Hence e dominates xn .
             x


54. lim ln x = lim xp = ∞.                                 59. If x → 0, then x2 → 0, so if lim
                                                                                                                        f (x)
                                                                                                                              = L,
    x→∞          x→∞
         ln x            ∞                                                                                        x→0   g(x)
     lim      is of type                                                 f (x2 )
    x→∞ xp               ∞                                     then lim          = L (but not conversely). If
    we use L’Hˆpital’s Rule to get
                o                                                    x→0 g(x2 )
            1
            x            1                                                             f (x)
     lim    p−1
                 = lim      = 0 (since p > 0).                 a = 0 or 1, then lim           involves the be-
    x→∞ px          x→∞ pxp                                                        x→a g(x)
                  p
    Therefore, x dominates ln x.                                                                         f (x2 )
                                                               havior of the quotient near a, while lim
             t                    t
                                                                                                    x→a g(x2 )
55. lim     e 2 − t3       Since e 2 dominates t3 .   So       involves the behavior of the quotient near the
    t→∞
             t                                                 different point a2 .
     lim    e − t3 = ∞
             2
    t→∞
                                                           60. Functions f (x) = |x| and g(x) = x work.
             √                                                      f (x)
             x − ln x            ∞                             lim        does not exist as it approaches −1
56. lim        √         is type   .                           x→0 g(x)
    x→∞          x               ∞
                                                               from the left and it approaches 1 from the
    we apply L’Hˆpital’s Rule to get
                  o
            √ − 1
            1                          √                                      f (x2 )
           2 x      x            x−2 x                         right, but lim         = 1.
     lim                = lim                                             x→0 g(x2 )
    x→∞        1
               √           x→∞       x
              2 x
                    2                                              2.5(4ωt − sin 4ωt)
    = lim 1 − √          = 1.                              61. lim
       x→∞            x                                        ω→0         4ω 2
                                                                      2.5(4t − 4t cos 4ωt)
                                                               = lim
         ln x3 + 2x + 1
                                                                 ω→0           8ω
57. lim                                                               2.5(16t2 sin 4ωt)
    x→∞ ln (x2 + x + 2)                                        = lim                    =0
                                                                 ω→0          8
    we apply L’Hˆpital’s Rule
                 o
            d
           dx  ln x3 + 2x + 1                                                                             π
     lim                                                                   2.5 − 2.5 sin(4ωt +              )
             d       2                                                                                    2 is type 0 ;
            dx (ln (x + x + 2))
    x→∞
                                                           62. lim                                                  0
                  3x2 +2
                                                               ω→0          4ω 2
                 x3 +2x+1                                      we apply L’Hˆpital’s Rule to get
                                                                            o
    = lim          2x+1                                            −10t cos(4ωt + π )
      x→∞                                                                          2
                 x2 +x+2                                       lim
              3x + 3x + 8x2 + 2x + 4
                    4     3
                                             3                 ω→0         8ω
    = lim                                  =                         40t2 sin(4ωt + π )
       x→∞     2x 4 + x3 + 4x2 + 4x + 1      2                 = lim                 2
                                                                                        = 5t2 .
    In general, for numerator and denominator the
                                                                 ω→0          8
    highest degee of polynomials p and q, such that
    p(x) > 0 and q(x) > 0 for x > 0,                              2

    should be the lim ln(p(x)) .
                          ln(q(x))
                       x→∞
                                                                 1.5


                 3x
         ln e + x           ∞
58. lim        2x + 4)
                         is   ;                                   1
    x→∞ ln (e               ∞
    we apply L’Hˆpital’s Rule
                   o                                             0.5
            d         3x
           dx ln e        +x
     lim     d
    x→∞
            dx  (ln (e2x + 4))                                    0
                                                                       0      0.1   0.2   0.3       0.4     0.5   0.6
                 3e3x +1                                                                        t
                 e3x +x
    = lim          2e2x
      x→∞
                  e2x +4
                    5x
                 3e    + 12e3x + e2x + 4        3          63. The area of triangular region 1 is
    = lim                                   =                  (1/2)(base)(height)
      x→∞              2e5x + 2xe2x             2
Ism et chapter_3
Ism et chapter_3
Ism et chapter_3
Ism et chapter_3
Ism et chapter_3
Ism et chapter_3
Ism et chapter_3
Ism et chapter_3
Ism et chapter_3
Ism et chapter_3
Ism et chapter_3
Ism et chapter_3
Ism et chapter_3
Ism et chapter_3
Ism et chapter_3
Ism et chapter_3
Ism et chapter_3
Ism et chapter_3
Ism et chapter_3
Ism et chapter_3
Ism et chapter_3
Ism et chapter_3
Ism et chapter_3
Ism et chapter_3
Ism et chapter_3
Ism et chapter_3
Ism et chapter_3
Ism et chapter_3
Ism et chapter_3
Ism et chapter_3
Ism et chapter_3
Ism et chapter_3
Ism et chapter_3
Ism et chapter_3
Ism et chapter_3
Ism et chapter_3
Ism et chapter_3
Ism et chapter_3
Ism et chapter_3
Ism et chapter_3
Ism et chapter_3
Ism et chapter_3
Ism et chapter_3
Ism et chapter_3
Ism et chapter_3
Ism et chapter_3
Ism et chapter_3
Ism et chapter_3
Ism et chapter_3
Ism et chapter_3
Ism et chapter_3
Ism et chapter_3
Ism et chapter_3
Ism et chapter_3
Ism et chapter_3
Ism et chapter_3
Ism et chapter_3
Ism et chapter_3
Ism et chapter_3
Ism et chapter_3
Ism et chapter_3
Ism et chapter_3
Ism et chapter_3
Ism et chapter_3
Ism et chapter_3
Ism et chapter_3
Ism et chapter_3
Ism et chapter_3
Ism et chapter_3
Ism et chapter_3
Ism et chapter_3
Ism et chapter_3
Ism et chapter_3
Ism et chapter_3
Ism et chapter_3
Ism et chapter_3

More Related Content

What's hot

Lesson 28: The Fundamental Theorem of Calculus
Lesson 28: The Fundamental Theorem of CalculusLesson 28: The Fundamental Theorem of Calculus
Lesson 28: The Fundamental Theorem of CalculusMatthew Leingang
 
Lesson 2: A Catalog of Essential Functions (slides)
Lesson 2: A Catalog of Essential Functions (slides)Lesson 2: A Catalog of Essential Functions (slides)
Lesson 2: A Catalog of Essential Functions (slides)Matthew Leingang
 
Profº Marcelo Santos Chaves Cálculo I (limites trigonométricos)
Profº Marcelo Santos Chaves   Cálculo I (limites trigonométricos)Profº Marcelo Santos Chaves   Cálculo I (limites trigonométricos)
Profº Marcelo Santos Chaves Cálculo I (limites trigonométricos)MarcelloSantosChaves
 
Lesson 4: Calcuating Limits (slides)
Lesson 4: Calcuating Limits (slides)Lesson 4: Calcuating Limits (slides)
Lesson 4: Calcuating Limits (slides)Matthew Leingang
 
Crib Sheet AP Calculus AB and BC exams
Crib Sheet AP Calculus AB and BC examsCrib Sheet AP Calculus AB and BC exams
Crib Sheet AP Calculus AB and BC examsA Jorge Garcia
 
Spm Add Maths Formula List Form4
Spm Add Maths Formula List Form4Spm Add Maths Formula List Form4
Spm Add Maths Formula List Form4guest76f49d
 
Form 4 formulae and note
Form 4 formulae and noteForm 4 formulae and note
Form 4 formulae and notesmktsj2
 
Quadratic functions and models
Quadratic functions and modelsQuadratic functions and models
Quadratic functions and modelsTarun Gehlot
 
237654933 mathematics-t-form-6
237654933 mathematics-t-form-6237654933 mathematics-t-form-6
237654933 mathematics-t-form-6homeworkping3
 
11 equations of planes
11 equations of planes11 equations of planes
11 equations of planesmath267
 
Lesson 16: Derivatives of Exponential and Logarithmic Functions
Lesson 16: Derivatives of Exponential and Logarithmic FunctionsLesson 16: Derivatives of Exponential and Logarithmic Functions
Lesson 16: Derivatives of Exponential and Logarithmic FunctionsMatthew Leingang
 
Linear algebra-solutions-manual-kuttler-1-30-11-otc
Linear algebra-solutions-manual-kuttler-1-30-11-otcLinear algebra-solutions-manual-kuttler-1-30-11-otc
Linear algebra-solutions-manual-kuttler-1-30-11-otckjalili
 

What's hot (18)

Cs 601
Cs 601Cs 601
Cs 601
 
Lesson 28: The Fundamental Theorem of Calculus
Lesson 28: The Fundamental Theorem of CalculusLesson 28: The Fundamental Theorem of Calculus
Lesson 28: The Fundamental Theorem of Calculus
 
Lesson 2: A Catalog of Essential Functions (slides)
Lesson 2: A Catalog of Essential Functions (slides)Lesson 2: A Catalog of Essential Functions (slides)
Lesson 2: A Catalog of Essential Functions (slides)
 
Profº Marcelo Santos Chaves Cálculo I (limites trigonométricos)
Profº Marcelo Santos Chaves   Cálculo I (limites trigonométricos)Profº Marcelo Santos Chaves   Cálculo I (limites trigonométricos)
Profº Marcelo Santos Chaves Cálculo I (limites trigonométricos)
 
Ex algebra (5)
Ex algebra  (5)Ex algebra  (5)
Ex algebra (5)
 
Lesson 4: Calcuating Limits (slides)
Lesson 4: Calcuating Limits (slides)Lesson 4: Calcuating Limits (slides)
Lesson 4: Calcuating Limits (slides)
 
Crib Sheet AP Calculus AB and BC exams
Crib Sheet AP Calculus AB and BC examsCrib Sheet AP Calculus AB and BC exams
Crib Sheet AP Calculus AB and BC exams
 
1203 ch 12 day 3
1203 ch 12 day 31203 ch 12 day 3
1203 ch 12 day 3
 
Spm Add Maths Formula List Form4
Spm Add Maths Formula List Form4Spm Add Maths Formula List Form4
Spm Add Maths Formula List Form4
 
1202 ch 12 day 2
1202 ch 12 day 21202 ch 12 day 2
1202 ch 12 day 2
 
1201 ch 12 day 1
1201 ch 12 day 11201 ch 12 day 1
1201 ch 12 day 1
 
Form 4 formulae and note
Form 4 formulae and noteForm 4 formulae and note
Form 4 formulae and note
 
Sect3 7
Sect3 7Sect3 7
Sect3 7
 
Quadratic functions and models
Quadratic functions and modelsQuadratic functions and models
Quadratic functions and models
 
237654933 mathematics-t-form-6
237654933 mathematics-t-form-6237654933 mathematics-t-form-6
237654933 mathematics-t-form-6
 
11 equations of planes
11 equations of planes11 equations of planes
11 equations of planes
 
Lesson 16: Derivatives of Exponential and Logarithmic Functions
Lesson 16: Derivatives of Exponential and Logarithmic FunctionsLesson 16: Derivatives of Exponential and Logarithmic Functions
Lesson 16: Derivatives of Exponential and Logarithmic Functions
 
Linear algebra-solutions-manual-kuttler-1-30-11-otc
Linear algebra-solutions-manual-kuttler-1-30-11-otcLinear algebra-solutions-manual-kuttler-1-30-11-otc
Linear algebra-solutions-manual-kuttler-1-30-11-otc
 

Similar to Ism et chapter_3

Calculus First Test 2011/10/20
Calculus First Test 2011/10/20Calculus First Test 2011/10/20
Calculus First Test 2011/10/20Kuan-Lun Wang
 
AP Calculus - Tutorial
AP Calculus - TutorialAP Calculus - Tutorial
AP Calculus - TutorialChris Wilson
 
Lesson 26: The Fundamental Theorem of Calculus (slides)
Lesson 26: The Fundamental Theorem of Calculus (slides)Lesson 26: The Fundamental Theorem of Calculus (slides)
Lesson 26: The Fundamental Theorem of Calculus (slides)Matthew Leingang
 
Lesson 26: The Fundamental Theorem of Calculus (slides)
Lesson 26: The Fundamental Theorem of Calculus (slides)Lesson 26: The Fundamental Theorem of Calculus (slides)
Lesson 26: The Fundamental Theorem of Calculus (slides)Matthew Leingang
 
Lesson 26: The Fundamental Theorem of Calculus (slides)
Lesson 26: The Fundamental Theorem of Calculus (slides)Lesson 26: The Fundamental Theorem of Calculus (slides)
Lesson 26: The Fundamental Theorem of Calculus (slides)Mel Anthony Pepito
 
2.2 Polynomial Function Notes
2.2 Polynomial Function Notes2.2 Polynomial Function Notes
2.2 Polynomial Function Noteslgemgnani
 
Lesson 21: Curve Sketching (slides)
Lesson 21: Curve Sketching (slides)Lesson 21: Curve Sketching (slides)
Lesson 21: Curve Sketching (slides)Mel Anthony Pepito
 
Fourier 3
Fourier 3Fourier 3
Fourier 3nugon
 
03 truncation errors
03 truncation errors03 truncation errors
03 truncation errorsmaheej
 
Mc ty-explogfns-2009-1
Mc ty-explogfns-2009-1Mc ty-explogfns-2009-1
Mc ty-explogfns-2009-1supoteta
 
limits and continuity
limits and continuitylimits and continuity
limits and continuityElias Dinsa
 
Bahan ajar kalkulus integral
Bahan ajar kalkulus integralBahan ajar kalkulus integral
Bahan ajar kalkulus integralgrand_livina_good
 
Interpolation functions
Interpolation functionsInterpolation functions
Interpolation functionsTarun Gehlot
 

Similar to Ism et chapter_3 (20)

Limits
LimitsLimits
Limits
 
Calculus First Test 2011/10/20
Calculus First Test 2011/10/20Calculus First Test 2011/10/20
Calculus First Test 2011/10/20
 
Imc2017 day2-solutions
Imc2017 day2-solutionsImc2017 day2-solutions
Imc2017 day2-solutions
 
Sect4 5
Sect4 5Sect4 5
Sect4 5
 
AP Calculus - Tutorial
AP Calculus - TutorialAP Calculus - Tutorial
AP Calculus - Tutorial
 
Lesson 26: The Fundamental Theorem of Calculus (slides)
Lesson 26: The Fundamental Theorem of Calculus (slides)Lesson 26: The Fundamental Theorem of Calculus (slides)
Lesson 26: The Fundamental Theorem of Calculus (slides)
 
Lesson 26: The Fundamental Theorem of Calculus (slides)
Lesson 26: The Fundamental Theorem of Calculus (slides)Lesson 26: The Fundamental Theorem of Calculus (slides)
Lesson 26: The Fundamental Theorem of Calculus (slides)
 
Lesson 26: The Fundamental Theorem of Calculus (slides)
Lesson 26: The Fundamental Theorem of Calculus (slides)Lesson 26: The Fundamental Theorem of Calculus (slides)
Lesson 26: The Fundamental Theorem of Calculus (slides)
 
2.2 Polynomial Function Notes
2.2 Polynomial Function Notes2.2 Polynomial Function Notes
2.2 Polynomial Function Notes
 
Lesson 21: Curve Sketching (slides)
Lesson 21: Curve Sketching (slides)Lesson 21: Curve Sketching (slides)
Lesson 21: Curve Sketching (slides)
 
Exponential functions
Exponential functionsExponential functions
Exponential functions
 
Fourier 3
Fourier 3Fourier 3
Fourier 3
 
03 truncation errors
03 truncation errors03 truncation errors
03 truncation errors
 
Mc ty-explogfns-2009-1
Mc ty-explogfns-2009-1Mc ty-explogfns-2009-1
Mc ty-explogfns-2009-1
 
parent functons
parent functonsparent functons
parent functons
 
Calculus Final Exam
Calculus Final ExamCalculus Final Exam
Calculus Final Exam
 
limits and continuity
limits and continuitylimits and continuity
limits and continuity
 
Bahan ajar kalkulus integral
Bahan ajar kalkulus integralBahan ajar kalkulus integral
Bahan ajar kalkulus integral
 
Sect1 2
Sect1 2Sect1 2
Sect1 2
 
Interpolation functions
Interpolation functionsInterpolation functions
Interpolation functions
 

More from Drradz Maths

More from Drradz Maths (20)

Figures
FiguresFigures
Figures
 
Formulas
FormulasFormulas
Formulas
 
Revision 7.1 7.3
Revision 7.1 7.3Revision 7.1 7.3
Revision 7.1 7.3
 
Tutorial 9
Tutorial 9Tutorial 9
Tutorial 9
 
Tutorial 8
Tutorial 8Tutorial 8
Tutorial 8
 
MTH3101 Tutor 7 lagrange multiplier
MTH3101  Tutor 7 lagrange multiplierMTH3101  Tutor 7 lagrange multiplier
MTH3101 Tutor 7 lagrange multiplier
 
Tutorial 6 en.mughti important
Tutorial 6 en.mughti importantTutorial 6 en.mughti important
Tutorial 6 en.mughti important
 
Figures
FiguresFigures
Figures
 
Formulas
FormulasFormulas
Formulas
 
Figures
FiguresFigures
Figures
 
First_attachment MTH3101
First_attachment MTH3101First_attachment MTH3101
First_attachment MTH3101
 
Tutorial 9 mth 3201
Tutorial 9 mth 3201Tutorial 9 mth 3201
Tutorial 9 mth 3201
 
Tutorial 8 mth 3201
Tutorial 8 mth 3201Tutorial 8 mth 3201
Tutorial 8 mth 3201
 
Tutorial 7 mth 3201
Tutorial 7 mth 3201Tutorial 7 mth 3201
Tutorial 7 mth 3201
 
Tutorial 6 mth 3201
Tutorial 6 mth 3201Tutorial 6 mth 3201
Tutorial 6 mth 3201
 
Tutorial 5 mth 3201
Tutorial 5 mth 3201Tutorial 5 mth 3201
Tutorial 5 mth 3201
 
Tutorial 4 mth 3201
Tutorial 4 mth 3201Tutorial 4 mth 3201
Tutorial 4 mth 3201
 
Tutorial 2, 2(e), no. 7
Tutorial 2,  2(e),  no. 7Tutorial 2,  2(e),  no. 7
Tutorial 2, 2(e), no. 7
 
Tutorial 3 mth 3201
Tutorial 3 mth 3201Tutorial 3 mth 3201
Tutorial 3 mth 3201
 
Tutorial 2 mth 3201
Tutorial 2 mth 3201Tutorial 2 mth 3201
Tutorial 2 mth 3201
 

Recently uploaded

Expert Pool Table Refelting in Lee & Collier County, FL
Expert Pool Table Refelting in Lee & Collier County, FLExpert Pool Table Refelting in Lee & Collier County, FL
Expert Pool Table Refelting in Lee & Collier County, FLAll American Billiards
 
IPL Quiz ( weekly quiz) by SJU quizzers.
IPL Quiz ( weekly quiz) by SJU quizzers.IPL Quiz ( weekly quiz) by SJU quizzers.
IPL Quiz ( weekly quiz) by SJU quizzers.SJU Quizzers
 
Instruction Manual | ThermTec Hunt Thermal Clip-On Series | Optics Trade
Instruction Manual | ThermTec Hunt Thermal Clip-On Series | Optics TradeInstruction Manual | ThermTec Hunt Thermal Clip-On Series | Optics Trade
Instruction Manual | ThermTec Hunt Thermal Clip-On Series | Optics TradeOptics-Trade
 
Austria vs France David Alaba Switches Position to Defender in Austria's Euro...
Austria vs France David Alaba Switches Position to Defender in Austria's Euro...Austria vs France David Alaba Switches Position to Defender in Austria's Euro...
Austria vs France David Alaba Switches Position to Defender in Austria's Euro...Eticketing.co
 
Italy Vs Albania Euro Cup 2024 Italy's Strategy for Success.docx
Italy Vs Albania Euro Cup 2024 Italy's Strategy for Success.docxItaly Vs Albania Euro Cup 2024 Italy's Strategy for Success.docx
Italy Vs Albania Euro Cup 2024 Italy's Strategy for Success.docxWorld Wide Tickets And Hospitality
 
Instruction Manual | ThermTec Wild Thermal Monoculars | Optics Trade
Instruction Manual | ThermTec Wild Thermal Monoculars | Optics TradeInstruction Manual | ThermTec Wild Thermal Monoculars | Optics Trade
Instruction Manual | ThermTec Wild Thermal Monoculars | Optics TradeOptics-Trade
 
Turkiye Vs Georgia Turkey's UEFA Euro 2024 Journey with High Hopes.pdf
Turkiye Vs Georgia Turkey's UEFA Euro 2024 Journey with High Hopes.pdfTurkiye Vs Georgia Turkey's UEFA Euro 2024 Journey with High Hopes.pdf
Turkiye Vs Georgia Turkey's UEFA Euro 2024 Journey with High Hopes.pdfEticketing.co
 
Technical Data | ThermTec Wild 335 | Optics Trade
Technical Data | ThermTec Wild 335 | Optics TradeTechnical Data | ThermTec Wild 335 | Optics Trade
Technical Data | ThermTec Wild 335 | Optics TradeOptics-Trade
 
France's UEFA Euro 2024 Ambitions Amid Coman's Injury.docx
France's UEFA Euro 2024 Ambitions Amid Coman's Injury.docxFrance's UEFA Euro 2024 Ambitions Amid Coman's Injury.docx
France's UEFA Euro 2024 Ambitions Amid Coman's Injury.docxEuro Cup 2024 Tickets
 
Austria VS France Injury Woes a Look at Euro 2024 Qualifiers.docx
Austria VS France Injury Woes a Look at Euro 2024 Qualifiers.docxAustria VS France Injury Woes a Look at Euro 2024 Qualifiers.docx
Austria VS France Injury Woes a Look at Euro 2024 Qualifiers.docxWorld Wide Tickets And Hospitality
 
Real Moto 2 MOD APK v1.1.721 All Bikes, Unlimited Money
Real Moto 2 MOD APK v1.1.721 All Bikes, Unlimited MoneyReal Moto 2 MOD APK v1.1.721 All Bikes, Unlimited Money
Real Moto 2 MOD APK v1.1.721 All Bikes, Unlimited MoneyApk Toly
 
Technical Data | ThermTec Wild 650L | Optics Trade
Technical Data | ThermTec Wild 650L | Optics TradeTechnical Data | ThermTec Wild 650L | Optics Trade
Technical Data | ThermTec Wild 650L | Optics TradeOptics-Trade
 
Mysore Call Girls 7001305949 WhatsApp Number 24x7 Best Services
Mysore Call Girls 7001305949 WhatsApp Number 24x7 Best ServicesMysore Call Girls 7001305949 WhatsApp Number 24x7 Best Services
Mysore Call Girls 7001305949 WhatsApp Number 24x7 Best Servicesnajka9823
 
8377087607 ☎, Cash On Delivery Call Girls Service In Hauz Khas Delhi Enjoy 24/7
8377087607 ☎, Cash On Delivery Call Girls Service In Hauz Khas Delhi Enjoy 24/78377087607 ☎, Cash On Delivery Call Girls Service In Hauz Khas Delhi Enjoy 24/7
8377087607 ☎, Cash On Delivery Call Girls Service In Hauz Khas Delhi Enjoy 24/7dollysharma2066
 
JORNADA 3 LIGA MURO 2024GHGHGHGHGHGH.pdf
JORNADA 3 LIGA MURO 2024GHGHGHGHGHGH.pdfJORNADA 3 LIGA MURO 2024GHGHGHGHGHGH.pdf
JORNADA 3 LIGA MURO 2024GHGHGHGHGHGH.pdfArturo Pacheco Alvarez
 

Recently uploaded (16)

Expert Pool Table Refelting in Lee & Collier County, FL
Expert Pool Table Refelting in Lee & Collier County, FLExpert Pool Table Refelting in Lee & Collier County, FL
Expert Pool Table Refelting in Lee & Collier County, FL
 
Denmark Vs Serbia Haaland Euro Cup CPR Drive Incident.docx
Denmark Vs Serbia Haaland Euro Cup CPR Drive Incident.docxDenmark Vs Serbia Haaland Euro Cup CPR Drive Incident.docx
Denmark Vs Serbia Haaland Euro Cup CPR Drive Incident.docx
 
IPL Quiz ( weekly quiz) by SJU quizzers.
IPL Quiz ( weekly quiz) by SJU quizzers.IPL Quiz ( weekly quiz) by SJU quizzers.
IPL Quiz ( weekly quiz) by SJU quizzers.
 
Instruction Manual | ThermTec Hunt Thermal Clip-On Series | Optics Trade
Instruction Manual | ThermTec Hunt Thermal Clip-On Series | Optics TradeInstruction Manual | ThermTec Hunt Thermal Clip-On Series | Optics Trade
Instruction Manual | ThermTec Hunt Thermal Clip-On Series | Optics Trade
 
Austria vs France David Alaba Switches Position to Defender in Austria's Euro...
Austria vs France David Alaba Switches Position to Defender in Austria's Euro...Austria vs France David Alaba Switches Position to Defender in Austria's Euro...
Austria vs France David Alaba Switches Position to Defender in Austria's Euro...
 
Italy Vs Albania Euro Cup 2024 Italy's Strategy for Success.docx
Italy Vs Albania Euro Cup 2024 Italy's Strategy for Success.docxItaly Vs Albania Euro Cup 2024 Italy's Strategy for Success.docx
Italy Vs Albania Euro Cup 2024 Italy's Strategy for Success.docx
 
Instruction Manual | ThermTec Wild Thermal Monoculars | Optics Trade
Instruction Manual | ThermTec Wild Thermal Monoculars | Optics TradeInstruction Manual | ThermTec Wild Thermal Monoculars | Optics Trade
Instruction Manual | ThermTec Wild Thermal Monoculars | Optics Trade
 
Turkiye Vs Georgia Turkey's UEFA Euro 2024 Journey with High Hopes.pdf
Turkiye Vs Georgia Turkey's UEFA Euro 2024 Journey with High Hopes.pdfTurkiye Vs Georgia Turkey's UEFA Euro 2024 Journey with High Hopes.pdf
Turkiye Vs Georgia Turkey's UEFA Euro 2024 Journey with High Hopes.pdf
 
Technical Data | ThermTec Wild 335 | Optics Trade
Technical Data | ThermTec Wild 335 | Optics TradeTechnical Data | ThermTec Wild 335 | Optics Trade
Technical Data | ThermTec Wild 335 | Optics Trade
 
France's UEFA Euro 2024 Ambitions Amid Coman's Injury.docx
France's UEFA Euro 2024 Ambitions Amid Coman's Injury.docxFrance's UEFA Euro 2024 Ambitions Amid Coman's Injury.docx
France's UEFA Euro 2024 Ambitions Amid Coman's Injury.docx
 
Austria VS France Injury Woes a Look at Euro 2024 Qualifiers.docx
Austria VS France Injury Woes a Look at Euro 2024 Qualifiers.docxAustria VS France Injury Woes a Look at Euro 2024 Qualifiers.docx
Austria VS France Injury Woes a Look at Euro 2024 Qualifiers.docx
 
Real Moto 2 MOD APK v1.1.721 All Bikes, Unlimited Money
Real Moto 2 MOD APK v1.1.721 All Bikes, Unlimited MoneyReal Moto 2 MOD APK v1.1.721 All Bikes, Unlimited Money
Real Moto 2 MOD APK v1.1.721 All Bikes, Unlimited Money
 
Technical Data | ThermTec Wild 650L | Optics Trade
Technical Data | ThermTec Wild 650L | Optics TradeTechnical Data | ThermTec Wild 650L | Optics Trade
Technical Data | ThermTec Wild 650L | Optics Trade
 
Mysore Call Girls 7001305949 WhatsApp Number 24x7 Best Services
Mysore Call Girls 7001305949 WhatsApp Number 24x7 Best ServicesMysore Call Girls 7001305949 WhatsApp Number 24x7 Best Services
Mysore Call Girls 7001305949 WhatsApp Number 24x7 Best Services
 
8377087607 ☎, Cash On Delivery Call Girls Service In Hauz Khas Delhi Enjoy 24/7
8377087607 ☎, Cash On Delivery Call Girls Service In Hauz Khas Delhi Enjoy 24/78377087607 ☎, Cash On Delivery Call Girls Service In Hauz Khas Delhi Enjoy 24/7
8377087607 ☎, Cash On Delivery Call Girls Service In Hauz Khas Delhi Enjoy 24/7
 
JORNADA 3 LIGA MURO 2024GHGHGHGHGHGH.pdf
JORNADA 3 LIGA MURO 2024GHGHGHGHGHGH.pdfJORNADA 3 LIGA MURO 2024GHGHGHGHGHGH.pdf
JORNADA 3 LIGA MURO 2024GHGHGHGHGHGH.pdf
 

Ism et chapter_3

  • 1. −1/2 1 f (x0 ) = f (0) = (2 · 0 + 9) = 3 So, L(x) = f (x0 ) + f (x0 ) (x − x0 ) 1 = 3 + (x − 0) 3 Chapter 3 =3+ x 1 3 (b) Using √ the approximation L(x) to esti- √ mate 8.8, we get 8.8 = f (−0.1) ≈ Applications of 1 L(−0.1) = 3 + (−0.1) = 3 − 0.033 = 3 2.967 Differentiation 4. (a) f (x) = 2 , x0 = 1 x f (x0 ) = f (1) = 2 2 f (x) = − 2 and so f (1) = −2 x 3.1 Linear Approximations The linear approximation is L(x) = 2 + (−2) (x − 1) and Newtons Method √ (b) Using the approximation L(x) to estimate 1. (a) f (x) = x, x0 = 1 2 2 √ , we get = f (0.99) ≈ L(0.99) = f (x0 ) = f (1) = 1 = 1 0.99 0.99 1 2 + (−2)(0.99 − 1) = 2.02 f (x) = x−1/2 2 1 5. (a) f (x) = sin 3x, x0 = 0 f (x0 ) = f (1) = f (x0 ) = sin(3 · 0) = 0 2 So, f (x) = 3 cos 3x L(x) = f (x0 ) + f (x0 ) (x − x0 ) f (x0 ) = f (0) = 3 cos(3 · 0) = 3 1 So, = 1 + (x − 1) L(x) = f (x0 ) + f (x0 ) (x − x0 ) 2 1 1 = 0 + 3 (x − 0) = + x 2 2 = 3x (b) √ Using the approximation L(x) to estimate √ (b) Using the approximation L(x) to esti- 1.2, we get 1.2 = f (1.2) ≈ L(1.2) = 1 1 mate sin(0.3), we get sin(0.3) = f (0.1) ≈ + (1.2) = 1.1 L(0.1) = 3(0.1) = 0.3 2 2 2. (a) f (x0 ) = f (0) = 1 and 6. (a) f (x) = sin x, x0 = π 1 −2/3 f (x0 ) = sin π = 0 f (x) = (x + 1) 3 f (x) = cos x 1 So, f (0) = f (x0 ) = f (π) = cos π = −1 3 The linear approximation is, The Linear approximation is, 1 1 L(x) = f (x0 ) + f (x0 ) (x − x0 ) L(x) = 1 + (x − 0) = 1 + x 3 3 = 0 + (−1) (x − π) = π − x (b) Using the approximation L(x) to estimate √ √ 3 3 (b) Using the approximation L(x) to esti- 1.2, we get 1.2 = f (0.2) ≈ L(0.2) = mate sin(3.0), we get sin(3.0) = f (3.0) ≈ 1 1 + (0.2) = 1.066 L(3.0) = π − 3.0 3 √ √ 4 3. (a) f (x) = 2x + 9, √0 = 0 x 7. (a) f (x) = √ 16 + x, x0 = 0 4 f (x0 ) = f (0) = 2 · 0+9 = 3 f (0) = 16 + 0 = 2 1 1 f (x) = (2x + 9) −1/2 ·2 f (x) = (16 + x)−3/4 2 4 1 1 = (2x + 9) −1/2 f (0) = (16 + 0)−3/4 = 4 32 150
  • 2. 3.1. LINEAR APPROXIMATIONS AND NEWTONS METHOD 151 L(x) = f (0) + f (0)(x − 0) 36 L(72) = 120 + (72 − 80) 1 20 =2+ x = 120 + 1.8(−8) 32 1 = 105.6 cans = 2 + (0.04) = 2.00125 32 168 − 120 1 (b) L(x) = f (100) + (x − 100) (b) L(0.08) = 2 + (0.08) = 2.0025 100 − 80 32 48 1 L(94) = 168 − (94 − 100) (c) L(0.16) = 2 + (0.16) = 2.005 20 32 = 168 − 2.4(−6) 8. (a) f (x) = sin x, x0 = 0 = 182.4 cans f (0) = 0 f (x) = cos x 142 − 128 f (0) = cos 0 = 1 11. (a) L(x) = f (200) + (x − 200) 220 − 200 L(x) = f (0) + f (0) (x − 0) 14 L(208) = 128 + (208 − 200) =0+1·x 20 L(0.1) = 0.1 = 128 + 0.7(8) = 133.6 (b) f (x) = sin x, x0 = π 142 − 136 √ 3 π 3 (b) L(x) = f (240) + (x − 240) f = 220 − 240 3 2 6 π π 1 L(232) = 136 − (232 − 240) f = cos = 20 3 3 2 = 136 − 0.3(−8) = 138.4 π π π L(x) = f +f x− √ 3 3 3 3 1 π 14 − 8 L(1) = + 1− ≈ 0.842 12. (a) L(x) = f (10) + (x − 10) 2 2 3 10 − 5 2π 6 (c) f (x) = sin x, x0 = L(8) = 14 + (−2) = 11.6 √ 3 5 2π 3 14 − 8 f = (b) L(x) = f (10) + (x − 10) 3 2 10 − 5 2π 2π 1 6 f = cos =− L(12) = 14 + (2) = 16.4 3 3 2 5 2π 2π 2π L(x) = f +f x− 13. f (x) = x3 + 3x2 − 1 = 0, x0 = 1 3 3 3 √ f (x) = 3x2 + 6x 3 1 2π = − x− 2√ 2 3 f (x0 ) 9 3 1 9 2π (a) x1 = x0 − L = − − ≈ 0.788 f (x0 ) 4 2 2 4 3 13 + 3 · 12 − 1 18 − 14 =1− 9. (a) L(x) = f (20) + (x − 20) 3 · 12 + 6 · 1 20 − 30 3 2 4 =1− = L(24) ≈ 18 − (24 − 20) 9 3 10 f (x1 ) = 18 − 0.4(4) x2 = x1 − f (x1 ) = 16.4 games 2 3 2 2 2 3 +3 3 −1 14 − 12 = − 3 2 2 2 (b) L(x) = f (40) + (x − 40) 3 3 +6 3 30 − 40 2 79 f (36) ≈ 12 − (36 − 40) = ≈ 0.5486 10 144 = 12 − 0.2(−4) (b) 0.53209 = 12.8 games 120 − 84 14. f (x) = x3 + 4x2 − x − 1, x0 = −1 10. (a) L(x) = f (80) + (x − 80) f (x) = 3x2 + 8x − 1 80 − 60
  • 3. 152 CHAPTER 3. APPLICATIONS OF DIFFERENTIATION f (x0 ) 30 (a) x1 = x0 − f (x0 ) 3 1 = −1 − =− 20 −6 2 f (x1 ) y x2 = x1 − f (x1 ) 10 1 0.375 =− − = −0.4117647 2 −4.25 0 −5.0 −2.5 0.0 2.5 5.0 (b) The root is x ≈ −0.4064206546. x −10 Start with x0 = −5 to find the root near −5: 15. f (x) = x4 − 3x2 + 1 = 0, x0 = 1 x1 = −4.718750, x2 = −4.686202, f (x) = 4x3 − 6x x3 = −4.6857796, x4 = −4.6857795 f (x0 ) (a) x1 = x0 − 18. From the graph, we see two roots: f (x0 ) 14 − 3 · 12 + 1 1 =1− = 15 4 · 13 − 6 · 1 2 10 f (x1 ) x2 = x1 − f (x1 ) 5 -1 0 1 2 3 4 1 4 1 2 0 1 2 −3 2 +1 = − 2 1 3 1 -5 4 2 −6 2 -10 5 = 8 -15 -20 (b) 0.61803 16. f (x) = x4 − 3x2 + 1, x0 = −1 f (xi ) Use xi+1 = xi − with f (x) = 4x3 − 6x f (xi ) f (x) = x4 − 4x3 + x2 − 1, and f (x) = 4x3 − 12x2 + 2x f (x0 ) Start with x0 = 4 to find the root below 4: (a) x1 = x0 − x1 = 3.791666667, x2 = 3.753630030, x3 = f (x0 ) −1 1 3.7524339, x4 = 3.752432297 = −1 − =− Start with x = −1 to find the root just above 2 2 f (x1 ) −1: x2 = x1 − x1 = −0.7222222222, f (x1 ) x2 = −0.5810217936, 1 0.3125 x3 = −0.5416512863, =− − = −0.625 2 2.5 x4 = −0.5387668233, x5 = −0.5387519962 (b) The root is x ≈ −0.6180339887. f (xi ) f (xi ) 17. Use xi+1 = xi − with 19. Use xi+1 = xi − with f (xi ) f (xi ) f (x) = x3 + 4x2 − 3x + 1, and f (x) = x5 + 3x3 + x − 1, and f (x) = 3x2 + 8x − 3 f (x) = 5x4 + 9x2 + 1
  • 4. 3.1. LINEAR APPROXIMATIONS AND NEWTONS METHOD 153 10 x1 = −0.644108, x2 = −0.636751 x3 = −0.636733, x4 = −0.636733 Start with x0 = 1.5 to find the root near 1.5: 5 x1 = 1.413799, x2 = 1.409634 x3 = 1.409624, x4 = 1.409624 0 −1.0 −0.5 0.0 0.5 1.0 22. Use xi+1 = xi − f (xii)) with f (x x f (x) = cos x2 − x, and y −5 f (x) = 2x sin x2 − 1 3 −10 2 Start with x0 = 0.5 to find the root near 0.5: y x1 = 0.526316, x2 = 0.525262, 1 x3 = 0.525261, x4 = 0.525261 0 f (xi ) -2 -1 0 1 2 20. Use xi+1 = xi − with x f (xi ) -1 f (x) = cos x − x, and f (x) = − sin x − 1 -2 5.0 Start with x0 = 1 to find the root between 0 and 1: 2.5 x1 = 0.8286590991, x2 = 0.8016918647, x3 = 0.8010710854, x4 = 0.8010707652 0.0 3 −5 −4 −3 −2 −1 0 1 2 3 4 5 x 2 y −2.5 y 1 −5.0 0 Start with x0 = 1 to find the root near 1: -2 -1 0 1 x 2 x1 = 0.750364, x2 = 0.739113, -1 x3 = 0.739085, x4 = 0.739085 -2 21. Use xi+1 = xi − f (xii)) with f (x f (x) = sin x − x2 + 1, and f (xi ) f (x) = cos x − 2x 23. Use xi+1 = xi − with f (xi ) 5.0 f (x) = ex + x, and f (x) = ex + 1 20 2.5 15 0.0 −5 −4 −3 −2 −1 0 1 2 3 4 5 y 10 x y −2.5 5 −5.0 0 −3 −2 −1 0 1 2 3 x Start with x0 = −0.5 to find the root near −5 −0.5: Start with x0 = −0.5 to find the root between
  • 5. 154 CHAPTER 3. APPLICATIONS OF DIFFERENTIATION 0 and -1: zeros of f ), Newton’s method will succeed. x1 = −0.566311, x2 = −0.567143 Which root is found depends on the starting x3 = −0.567143, x4 = −0.567143 place. f (xi ) 33. f (x) = x2 + 1, x0 = 0 24. Use xi+1 = xi − with √ f (xi ) f (x) = 2x f (x) = e−x − x, and f (x0 ) 1 1 x1 = x0 − =0− f (x) = −e−x − √ f (x0 ) 0 2 x The method fails because f (x0 ) = 0. There are no roots. 1 34. Newton’s method fails because the function 0.5 does not have a root! 4x2 − 8x + 1 0 0 0.5 1 1.5 2 35. f (x) = = 0, x0 = −1 4x2 − 3x − 7 Note: f (x0 ) = f (−1) is undefined, so New- -0.5 ton’s Method fails because x0 is not in the do- main of f . Notice that f (x) = 0 only when -1 4x2 − 8x + 1 = 0. So using Newton’s Method on g(x) = 4x2 − 8x + 1 with x0 = −1 leads to x ≈ .1339. The other root is x ≈ 1.8660. Start with x0 = 0.5 to find the root close to 36. The slope tends to infinity at the zero. For ev- 0.5: ery starting point, the sequence does not con- x1 = 0.4234369253, x2 = 0.4262982542, verge. x3 = 0.4263027510 √ 37. (a) With x0 = 1.2, 25. f (x) = x2 − 11; x0 = 3; 11 ≈ 3.316625 √ x1 = 0.800000000, 26. Newton’s method for x near x = 23 is xn+1 = x2 = 0.950000000, 1 2 (xn + 23/xn ). Start with x0 = 5 to get: x3 = 0.995652174, x1 = 4.8, x2 = 4.7958333, and x3 = 4.7958315. x4 = 0.999962680, √ x5 = 0.999999997, 27. f (x) = x3 − 11; x0 = 2; 3 11 ≈ 2.22398 x6 = 1.000000000, √ x7 = 1.000000000 28. Newton’s method for 3 x near x = 23 is xn+1 = 1 (2xn + 23/x2 ). Start with x0 = 3 3 n (b) With x0 = 2.2, to get: x0 = 2.200000, x1 = 2.107692, x1 = 2.851851851, x2 = 2.843889316, and x2 = 2.056342, x3 = 2.028903, x3 = 2.884386698 x4 = 2.014652, x5 = 2.007378, √ x6 = 2.003703, x7 = 2.001855, 29. f (x) = x4.4 − 24; x0 = 2; 4.4 24 ≈ 2.059133 x8 = 2.000928, x9 = 2.000464, √ 30. Newton’s method for 4.6 x near x = 24 is x10 = 2.000232, x11 = 2.000116, 1 xn+1 = 4.6 (3.6xn +24/x3.6 ). Start with x0 = 2 n x12 = 2.000058, x13 = 2.000029, to get: x14 = 2.000015, x15 = 2.000007, x1 = 1.995417100, x2 = 1.995473305, and x16 = 2.000004, x17 = 2.000002, x3 = 1.995473304 x18 = 2.000001, x19 = 2.000000, x20 = 2.000000 31. f (x) = 4x3 − 7x2 + 1 = 0, x0 = 0 The convergence is much faster with x0 = f (x) = 12x2 − 14x 1.2. f (x0 ) 1 x1 = x0 − =0− f (x0 ) 0 38. Starting with x0 = 0.2 we get a sequence that The method fails because f (x0 ) = 0. Roots converges to 0 very slowly. (The 20th itera- are 0.3454, 0.4362, 1.659. tion is still not accurate past 7 decimal places). Starting with x0 = 3 we get a sequence that 32. Newton’s method fails because f = 0. As long 7 quickly converges to π. (The third iteration is as the sequence avoids xn = 0 and xn = (the already accurate to 10 decimal places!) 6
  • 6. 3.1. LINEAR APPROXIMATIONS AND NEWTONS METHOD 155 √ 39. (a) With x0 = −1.1 43. f (x) = √ 4 + x x1 = −1.0507937, f (0) = 4 + 0 = 2 x2 = −1.0256065, 1 f (x) = (4 + x)−1/2 x3 = −1.0128572, 2 1 1 x4 = −1.0064423, f (0) = (4 + 0)−1/2 = x5 = −1.0032246, 2 4 1 x6 = −1.0016132, L(x) = f (0) + f (0)(x − 0) = 2 + x 4 x7 = −1.0008068, 1 x8 = −1.0004035, L(0.01) = 2 + (0.01) = 2.0025 √ 4 x9 = −1.0002017, f (0.01) = 4 + 0.01 ≈ 2.002498 x10 = −1.0001009, 1 L(0.1) = 2 + (0.1) = 2.025 x11 = −1.0000504, √ 4 x12 = −1.0000252, f (0.1) = 4 + 0.1 ≈ 2.0248 x13 = −1.0000126, 1 L(1) = 2 + (1) = 2.25 x14 = −1.0000063, √ 4 x15 = −1.0000032, f (1) = 4 + 1 ≈ 2.2361 x16 = −1.0000016, x17 = −1.0000008, x18 = −1.0000004, x19 = −1.0000002, x20 = −1.0000001, 44. The linear approximation for ex at x = 0 is x21 = −1.0000000, L(x) = 1 + x. The error when x = 0.01 is x22 = −1.0000000 0.0000502, when x = 0.1 is 0.00517, and when (b) With x0 = 2.1 x = 1 is 0.718. x0 = 2.100000000, x1 = 2.006060606, x2 = 2.000024340, x3 = 2.000000000, x4 = 2.000000000 45. (a) f (0) = g(0) = h(0) = 1, so all pass The rate of convergence in (a) is slower through the point (0, 1). than the rate of convergence in (b). f (0) = 2(0 + 1) = 2, g (0) = 2 cos(2 · 0) = 2, and 40. From exercise 37, f (x) = (x − 1)(x − 2)2 . New- h (0) = 2e2·0 = 2, ton’s method converges slowly near the double so all have slope 2 at x = 0. root. From exercise 39, f (x) = (x − 2)(x + 1)2 . The linear approximation at x = 0 for all Newton’s method again converges slowly near three functions is L(x) = 1 + 2x. the double root. In exercise 38, Newton’s method converges slowly near 0, which is a zero of both x and sin x but converges quickly near π, which is a zero only of sin x. (b) Graph of f (x) = (x + 1)2 : 5 41. f (x) = tan x, f (0) = tan 0 = 0 f (x) = sec2 x, f (0) = sec2 0 = 1 4 L(x) = f (0) + f (0)(x − 0) L(0.01) = 0.01 3 = 0 + 1(x − 0) = x y f (0.01) = tan 0.01 ≈ 0.0100003 2 L(0.1) = 0.1 1 f (0.1) = tan(0.1) ≈ 0.1003 L(1) = 1 0 f (1) = tan 1 ≈ 1.557 −3 −2 −1 0 1 2 3 −1 √ x 42. The linear approximation for 1 + x at x = 0 1 is L(x) = 1 + 2 x. The error when x = 0.01 is 0.0000124, when x = 0.1 is 0.00119, and when x = 1 is 0.0858. Graph of f (x) = 1 + sin(2x):
  • 7. 156 CHAPTER 3. APPLICATIONS OF DIFFERENTIATION 5 2 4 1 3 y 2 0 -2 -1 0 1 2 x 1 -1 0 −3 −2 −1 0 1 2 3 -2 x −1 Graph of h(x) = sinh x: Graph of f (x) = e2x : 3 5 2 4 1 3 0 -2 -1 0 1 2 y -1 x 2 -2 1 -3 0 −3 −2 −1 0 1 2 3 x −1 sin x is the closest fit, but sinh x is close. √ 4 47. (a) 16.04 = 2.0012488 L(0.04) = 2.00125 |2.0012488 − 2.00125| = .00000117 46. (a) f (0) = g(0) = h(0) = 0, so all pass √ 4 through the point (0, 0). (b) 16.08 = 2.0024953 f (0) = cos 0 = 1, L(.08) = 2.0025 1 |2.0024953 − 2.0025| = .00000467 g (0) = = 1, and 1 + 02 √ h (0) = cosh 0 = 1, (c) 4 16.16 = 2.0049814 so all have slope 1 at x = 0. L(.16) = 2.005 The linear approximation at x = 0 for all |2.0049814 − 2.005| = .0000186 three functions is L(x) = x. (b) Graph of f (x) = sin x: 48. If you graph | tan x − x|, you see that the dif- ference is less than .01 on the interval −.306 < 2 x < .306 (In fact, a slightly larger interval would work as well). 1 49. The first tangent line intersects the x-axis at a 0 -2 -1 0 1 2 point a little to the right of 1. So x1 is about x 1.25 (very roughly). The second tangent line -1 intersects the x-axis at a point between 1 and x1 , so x2 is about 1.1 (very roughly). Newton’s -2 Method will converge to the zero at x = 1. Starting with x0 = −2, Newton’s method con- Graph of g(x) = tan−1 x: verges to x = −1.
  • 8. 3.1. LINEAR APPROXIMATIONS AND NEWTONS METHOD 157 f (x) = 2x − 1 3 3 At x0 = 2 2 2 y 3 3 1 f (x0 ) = − −1=− 1 2 2 4 and 3 -2 -1 0 0 1 2 f (x0 ) = 2 −1=2 x 2 -1 By Newton’s formula, f (x0 ) 3 −1 13 x1 = x0 − = − 4 = -2 f (x0 ) 2 2 8 Starting with x0 = 0.4, Newton’s method con- (b) f (x) = x2 − x − 1 verges to x = 1. f (x) = 2x − 1 5 At x0 = 3 3 2 5 5 1 f (x0 ) = − −1= 2 3 3 9 y and 5 7 1 f (x0 ) = 2 −1= 3 3 0 By Newton’s formula, -2 -1 0 1 2 f (x0 ) x x1 = x0 − -1 f (x0 ) 1 5 9 5 1 34 -2 = − 7 = − = 3 3 3 21 21 50. It wouldn’t work because f (0) = 0. x0 = 0.2 (c) f (x) = x2 − x − 1 works better as an initial guess. After jumping f (x) = 2x − 1 8 to x1 = 2.55, the sequence rapidly decreases At x0 = 5 2 toward x = 1. Starting with x0 = 10, it takes 8 8 1 f (x0 ) = − −1=− several steps to get to 2.5, on the way to x = 1. 5 5 25 and f (xn ) 8 11 51. xn+1 = xn − f (x0 ) = 2 −1= f (xn ) 5 5 x2 − c n By Newton’s formula, = xn − f (x0 ) 2xn x1 = x0 − x2 c f (x0 ) = xn − n + 8 − 25 1 8 1 89 2xn 2xn = − 11 = + = xn c 5 5 55 55 = + 5 2 2xn 1 c (d) From part (a), = xn + F4 F7 2 xn sincex0 = , hence x1 = . √ √ √ F3 F6 If x0 < a, then a/x0 > a, so x0 < a < From part (b), a/x0 . F5 F9 √ since x0 = hence x1 = . 52. The root of xn − c is n c, so Newton’s method F4 F8 From part (c), approximates this number. F6 F11 Newton’s method gives since x0 = hence x1 = . f (xi ) xn − c F5 F10 xi+1 = xi − = xi − i n−1 Fn+1 f (xi ) nxi Thus in general if x0 = , then x1 = Fn 1 F2n+1 = (nxi − xi + cx1−n ), i implies m = 2n + 1 and k = 2n n F2n as desired. 3 Fn+1 53. (a) f (x) = x2 − x − 1 (e) Given x0 = , then lim will be 2 n→∞ Fn
  • 9. 158 CHAPTER 3. APPLICATIONS OF DIFFERENTIATION the zero of the function f (x) = x2 − 2P x L(x) = 120 − .01(120) = P − x − 1 which is 1.618034. Therefore, R Fn+1 2 · 120x lim = 1.618034 = 120 − n→∞ Fn R 2x .01 = 54. The general form of functionf (x) is, R 1 n+2 1 1 x = .005R = .005(20,900,000) fn (x) = 2 x − 3 for n < x < n−1 . 5 2 2 = 104,500 ft Hence 2n+2 1 1 58. If m = m0 (1 − v 2 /c2 )1/2 , then f (x) = fn (x) = for n < x < n−1 . 5 2 2 m = (m0 /2)(1 − v 2 /c2 )−1/2 (−2v/c2 ), and By Newton’s method, m = 0 when v = 0. The linear approxima- 3 f 34 3 f1 34 x1 = − = − tion is the constant function m = m0 for small 4 f 3 4 4 f1 4 3 values v. 3 (3/5 ) 3 x0 = − = = 59. The only positive solution is 0.6407. 4 (8/5 ) 8 2 x1 x0 x0 Similarly, x2 = = 2 and x3 = 3 60. The smallest positive solution of the first equa- 2 2 2 tion is 0.132782, and for the second equa- x0 Continuing this, we get, xn−1 = n−1 It may tion the smallest positive solution is 1, so the 2 also be observed that, for each fn (x) species modeled by the second equation is cer- (1/2n ) + 1/2n+1 3 tain to go extinct. This is consistent with the x0 = = n+1 , 2 2 models, since the expected number of offspring x0 3 3 for the population modeled by the first equa- xn = n = 2n+1 ⇒ xn+1 = 2n+2 which 2 2 2 tion is 2.2, while for the second equation it is is the zero of F . Therefore Newton’s method only 1.3 converges to zero of F . 61. The linear approximation for the inverse tan- 55. For small x we approximate ex by x + 1 gent function at x = 0 is (see exercise 44) f (x) ≈ f (0) + f (0)(x − 0) Le2πd/L − e−2πd/L tan−1 (x) ≈ tan−1 (0) + 1+02 (x − 0) 1 e2πd/L + e−2πd/L tan−1 (x) ≈ x L 1 + 2πd − 1 − 2πd L L Using this approximation, ≈ 3[1 − d/D] − w/2 1 + 2πd + 1 − 2πd L L φ = tan−1 4πd D−d L L ≈ = 2πd 3[1 − d/D] − w/2 2 φ≈ 4.9 D−d f (d) ≈ · 2πd = 9.8d If d = 0, then φ ≈ 3−w/2 . Thus, if w or D π D increase, then φ decreases. 8πhcx−5 56. If f (x) = , then using the linear 62. d (θ) = D(w/6 sin θ) ehc/(kT x) − 1 d(0) = D(1 − w/6) so approximation we see that 8πhcx−5 d(θ) ≈ d(0) + d (0)(θ − 0) f (x) ≈ hc = 8πkT x−4 = D(1 − w/6) + 0(θ) = D(1 − w/6), (1 + kT x ) − 1 as desired. as desired. P R2 57. W (x) = (R + x)2 , x0 = 0 3.2 Indeterminate Forms and W (x) = −2P R2 L’Hˆpital’s Rule o (R + x)3 L(x) = W (x0 ) + W (x0 )(x − x0 ) x+2 1. lim x→−2 x2 − 4 P R2 −2P R2 x+2 = + (x − 0) = lim (R + 0)2 (R + 0)3 x→−2 (x + 2)(x − 2) 2P x 1 1 =P− = lim =− R x→−2 x − 2 4
  • 10. ˆ 3.2. INDETERMINATE FORMS AND L’HOPITAL’S RULE 159 x2 − 4 sin x − x 0 2. lim 11. lim 3 is type ; x→2 x2− 3x + 2 x→0 x 0 (x − 2)(x + 2) we apply L’Hˆpital’s Rule thrice to get o = lim cos x − 1 − sin x x→2 (x − 2)(x − 1) = lim = lim x+2 x→0 3x2 x→0 6x = lim =4 − cos x 1 x→2 x − 1 = lim =− . x→0 6 6 3x2 + 2 3. lim tan x − x 0 x→∞ x2 − 4 12. lim is type ; 2 3 + x2 x→0 x3 0 = lim we apply L’Hˆpital’s Rule to get o x→∞ 1 − 4 sec2 x − 1 x2 3 lim . = =3 x→0 3x2 1 Apply L’Hˆpital’s Rule twice more to get o x+1 ∞ 2 sec2 x tan x 4. lim is type ; lim x→−∞ x2 + 4x + 3 ∞ x→0 6x 4 sec2 x tan2 x + 2 sec4 x 1 we apply L’Hˆpital’s Rule to get o = lim = . 1 x→0 6 3 lim = 0. √ √ √ x→−∞ 2x + 4 t−1 t−1 t+1 13. lim = lim · √ e2t − 1 0 t→1 t − 1 t→1 t − 1 t+1 5. lim is type ; t→0 t 0 (t − 1) we apply L’Hˆpital’s Rule to get o = lim √ t→1 (t − 1) t + 1 d e2t − 1 lim dt d 1 1 = lim √ = dt t t→0 t→1 t+1 2 2e2t 2 lim = =2 ln t 0 t→0 1 1 14. lim is type ; sin t 0 t→1 t −1 0 6. lim is type ; t→0 e3t−1 0 we apply L’Hˆpital’s Rule to get o we apply L’Hˆpital’s Rule to get o d 1 dt (ln t) d (sin t) cos t 1 lim d = lim t = 1 t→1 1 dt (t − 1) t→1 lim ddt 3t = lim 3t = t→0 3e 3 dt (e − 1) t→0 x3 ∞ tan−1 t 0 15. lim x is type ; 7. lim is type ; x→∞ e ∞ t→0 sin t 0 we apply L’Hˆpital’s Rule thrice to get o we apply L’Hˆpital’s Rule to get o d tan−1 t 1/(1 + t2 ) 3x2 6x lim dt d = lim =1 lim = lim x t→0 dt (sin t) t→0 cos t x→∞ ex x→∞ e 6 sin t 0 = lim x = 0. 8. lim is type ; x→∞ e t→0 sin−1 t 0 ex ∞ we apply L’Hˆpital’s Rule to get o 16. lim is type ; d x→∞ x4 ∞ dt (sin t) cos t lim = lim √ =1 we apply L’Hˆpital’s Rule four times to get o t→0 d sin−1 t t→0 1/( 1 − t2 ) dt ex ex lim 3 = lim sin 2x 0 x→∞ 4x x→∞ 12x2 9. lim is type ; ex ex x→π sin x 0 = lim = lim = ∞. x→∞ 24x x→∞ 24 we apply L’Hˆpital’s Rule to get o x cos x − sin x ∞ 2 cos 2x 2(1) 17. limx→0 2 is type ; lim = = −2. x sin x ∞ x→π cos x −1 we apply L’Hˆpital’s Rule twice to get o cos x − x sin x − cos x cos−1 x limx→0 10. lim is undefined (numerator goes to sin2 x + 2x sin x cos x x→−1 x2 − 1 −x sin x π, denominator goes to 0). = lim x→0 sin x (sin x + 2x cos x)
  • 11. 160 CHAPTER 3. APPLICATIONS OF DIFFERENTIATION −x x − π cos x = lim = lim 2 = 0 x→0 sin x + 2x cos x −1 π x→ 2 cos x − x − π sin x 2 = lim x→0 cos x + 2 cos x − 2x sin x ln x ∞ 1 21. lim 2 is type =− . x→∞ x ∞ 3 we apply L’Hˆpital’s Rule to get o 1/x 1 18. Rewrite as one fraction, we have lim = lim = 0. x→∞ 2x x→∞ 2x2 1 x cos x − sin x lim cot x − = lim ln x ∞ x→0 x x→0 x sin x 22. lim √ is type ; 0 x→∞ x ∞ which is of type we apply L’Hˆpital’s Rule to get o 0 1 we apply L’Hˆpital’s Rule to get o 2 cos x − x sin x − cos x lim x = lim √ = 0. 1 = lim x→∞ √ 2 x x→∞ x x→0 sin x + x cos x t ∞ d (−x sin x) 23. lim t is type = lim dx t→∞ e ∞ x→0 d (sin x + x cos x) we apply L’Hˆpital’s Rule to get o dx d (t) 1 − sin x − x cos x lim dt = lim t = 0. = lim =0 t→∞ d (et ) t→∞ e x→0 cos x + cos x − x sin x dt sin 1 t 0 19. Rewrite as one fraction, we have 24. lim 1 is type t→∞ t 0 x+1 2 we apply L’Hˆpital’s Rule to get o lim − x→0 x sin 2x - 1 cos 1 2 1 (x + 1) sin 2x − 2x 0 = lim t 1 t = lim cos = 1. = lim is type ; t→∞ − t2 t→∞ t x→0 x sin 2x 0 we apply L’Hˆpital’s Rule four times to get o ln (ln t) d 25. lim (x + 1) sin 2x − 2x t→1 ln t lim dx d As t approaches ln from below, ln t is a small dx (x sin 2x) x→0 negative number. Hence ln (ln t) is undefined, sin 2x + 2(x + 1) cos 2x − 2 so the limit is undefined. = lim x→0 sin 2x + 2x cos 2x d sin (sin t) 0 (sin 2x + 2(x + 1) cos 2x − 2) 26. lim is type = lim dx d t→0 sin t 0 dx (sin 2x + 2x cos 2x) x→0 we apply L’Hˆpital’s Rule to get o 2 cos 2x + 2 cos 2x − 4(x + 1) sin 2x cos (sin t) cos t = lim lim = 1. x→0 2 cos 2x + 2 cos 2x − 4x sin 2x t→0 cos t 4 = =1 sin (sinh x) 0 4 27. lim is type x→0 sinh (sin x) 0 we apply L’Hˆpital’s Rule to get o 1 cos (sinh x) cosh x 20. lim tan x + lim =1 π x→ 2 x− π 2 x→0 cosh (sin x) cos x In this case the limit has the form (∞ - ∞). sin x − sinh x sin x 28. lim Rewrite tan x as and then as one frac- x→0 cos x − cosh x cos x tion, we get 2 sin x − ex + e−x = lim 1 x→0 2 cos x − ex − e−x lim tan x + x→ 2π x− π 2 2ex sin x − e2x + 1 0 = lim is type sin x 1 x→0 2ex cos x − e2x − 1 0 = lim + we apply L’Hˆpital’s Rule twice to get o x→ π 2 cos x x − π 2 2ex cos x + 2ex sin x − 2e2x x − π sin x + cos x 2 0 lim = lim is type x→0 −2ex sin x + 2ex cos x − 2e2x x→ π 2 x − π cos x 2 0 cos x + sin x − 1 0 we apply L’Hˆpital’s Rule to get o = lim is type x→0 cos x − sin x − 1 0 sin x + x − π cos x − sin x 2 − sin x + cos x = lim = lim = −1 x→ π 2 cos x − x − π sin x 2 x→0 − sin x − cos x
  • 12. ˆ 3.2. INDETERMINATE FORMS AND L’HOPITAL’S RULE 161 ln x ∞ x+1   29. lim is type x→0 + cot x ∞  ln x−2  we apply L’Hˆpital’s Rule to get o = lim   x→∞  √ 1  1/x x2 −4 lim x→0+ − csc2 x 0 sin x This last limit has indeterminate form , so = lim+ − sin x · = (0)(1) = 0. 0 x→0 x we can apply L’Hˆpital’s Rule and simplify to o find that the above is equal to √ x −3(x2 − 4)3/2 30. lim+ = 0 (numerator goes to 0 and de- lim and this is equal to 3. So x→0 ln x x→∞ −x3 + x2 + 2x nominator goes to −∞). lim ln y = 3. x→∞ Thus lim y = lim eln y = e3 ≈ 20.086. x→∞ x→∞ 31. lim x2 + 1 − x √ x→∞ √ 1 x x2 + 1 + x 35. lim+ √ −√ = lim x 2+1−x √ x→0 x√ x+1 √ x→∞ x2 + 1 + x x + 1 − ( x)2 2 x +1−x 2 = lim+ √ √ = lim √ x→0 √ x x+1 x→∞ x2 + 1 + x x+1−x 1 = lim √ √ = lim √ =0 x→0+ x x+1 x→∞ x 2+1+x = ∞. √ 5−x−2 0 ln x −1 36. lim √ is type 32. lim ln x − x = lim x = −∞ since the x→1 10 − x − 3 0 1 x→∞ x→∞ x we apply L’Hˆpital’s Rule to get o numerator goes to −1 and the denominator 1 −1/2 2 (5 − x) (−1) goes to 0+ . (Recall Example 2.8 which shows lim 1 x→1 (10 − x)−1/2 (−1) 2 √ ln x lim = 0.) 10 − x 3 x→∞ x = lim √ = . x→1 5−x 2 x x 1 37. Let y = (1/x) . Then ln y = x ln(1/x). Then 33. Let y = 1+ x lim+ ln y = lim+ x ln(1/x) = 0, by Exercise x→0 x→0 1 ⇒ ln y = x ln 1 + . Then 27. Thus lim+ y = lim+ eln y = 1. x x→0 x→0 1 lim ln y = lim x ln 1 + 38. Let y = lim+ (cos x)1/x . Then x→∞ x→∞ x x→0 ln 1 + x 1 1 = lim ln y = lim+ ln cos x x→0 x x→∞ 1/x 1 1 ln(cos x) 0 1+ x1 − x2 = lim is type = lim x→0 + x 0 x→∞ −1/x2 so apply L’Hˆpital’s Rule to get o 1 − tan x = lim = 1. lim+ = 0. x→∞ 1 + 1 x→0 1 x Hence lim y = lim eln y = e. Therefore the limit is y = e0 = 1. x→∞ x→∞ t t t−3 (t − 3) 39. lim = lim 34. Notice that the limit in question has the inde- t→∞ t+2 t→∞ (t + 2)t terminate form 1∞ . Also, note that as x gets 3 t lim 1 − 3 t x+1 x+1 1− t t→∞ t large, = . = lim = t→∞ 2 t 2 t x−2 x −√2 1+ t lim 1 + t t→∞ x2 −4 −3 t x+1 lim 1 + Define y = . Then t→∞ t e−3 x−2 = = = e−5 2 t e2 √ x+1 lim 1 + t t→∞ ln y = x2 − 4 ln and x−2 t t 3 x+1 t−3 1− t lim ln y = lim x2 − 4 ln 40. lim = lim 1 x→∞ x→∞ x−2 t→∞ 2t + 1 t→∞ 2+ t
  • 13. 162 CHAPTER 3. APPLICATIONS OF DIFFERENTIATION 3 t 1− t e−3 we apply L’Hˆpital’s Rule to get o = lim t = lim =0 t→∞ 1/2 t→∞ 2t e1/2 2t 1 + n cos nx n t lim = . x→0 m cos mx m 41. L’Hˆpital’s rule does not apply. As x → 0, the o sin x2 2x cos x2 numerator gets close to 1 and the denominator 50. (a) lim 2 = lim x→0 x x→0 2x is small and positive. Hence the limit is ∞. = lim cos x2 = 1, ex − 1 0 ex x→0 42. lim is type , but lim is not, so sin x x→0 x 2 0 x→0 2x which is the same as lim . L’Hˆpital’s Rule does not apply to this limit. o x→0 x 1 − cos x2 43. L’Hˆpital’s rule does not apply. As x → 0, the o (b) lim x→0 x4 numerator is small and positive while the de- 2x sin x2 sin x2 nominator goes to −∞. Hence the limit is 0. = lim 3 = lim x→0 4x x→0 2x2 2x 1 sin x 2 1 Also lim , which equals lim x2 , is not of = lim = (by part (a)), x→0 2/x x→0 2 x→0 x2 2 0 the form so L’Hˆpital’s rule doesn’t apply o 0 while here either. sin x 0 cos x 1 − cos x sin x 1 1 44. lim is type , but lim is not, so lim 2 = lim = (1) = x→0 x2 0 x→0 2x x→0 x x→0 2x 2 2 L’Hˆpital’s rule does not apply. This limit is o so both of these limits are the same. undefined because the numerator goes to 1 and (c) Based on the patterns found in exercise the denominator goes to 0. 45, we should guess csc x sin x3 1 − cos x3 1 45. lim+ √ lim = 1 and lim = . x→0 x x→0 x3 x→0 x6 2 ∞ In this case limit has the form , L’Hˆspital’s o 0 (x + 1)(2 + sin x) Rule should not be used. 51. (a) x(2 + cos x) x−3/2 ∞ x 46. lim+ is type . In this case (b) x x→0 ln x −∞ e L’Hˆspital’s Rule should be used. o 3x + 1 (c) x2 − 3x + 1 x−7 47. lim = ∞. In this case limit has 3 − 8x x→∞ tan−1 x (d) the form ∞. So L’Hˆspital’s Rule should not o 1 + 2x be used. 52. (a) lim x − ln x = ∞ (see exercise 32). ln x2 ∞ x→∞ 48. lim is type . So L’Hˆspital’s Rule o √ x→∞ ex/3 ∞ (b) lim x2 + 1 − x = 0 (see exercise 31). should be used. x→∞ √ sin 3x (c) lim x2 + 4x − x 49. (a) Starting with lim , we cannot x→∞ √ sin 2x x→0 = lim ( x2 + 4x − x) 3x x→∞ “cancel sin”to get lim . We can cancel 4x x→0 2x = lim √ the x’s in the last limit to get the final an- x→∞ x2 + 4x + x swser of 3/2. The first step is likely to give 1 4x x a correct answer because the linear ap- = lim √ x→∞ 1 proximation of sin 3x is 3x, and the linear ( x2 + 4x + x) x approximation of sin 2x is 2x. The linear 4 = lim = 2, approximations are better the closer x is x→∞ 4 1+ x +1 to zero, so the limits are likely to be the where to get from the second to same. the third line, we have multiplied by √ sin nx (b) lim is type 0 ; 0 ( x2 + 4x + x) x→0 sin mx √ . ( x2 + 4x + x)
  • 14. ˆ 3.2. INDETERMINATE FORMS AND L’HOPITAL’S RULE 163 53. lim ex = lim xn = ∞ In general,when the degree of exponential term x→∞ x→∞ ex in the numerator and denominator are differ- lim n = ∞. Since n applications of ln ekx + p(x) x→∞ x ent, then the lim for polyno- L’Hˆpital’s rule yields o x→∞ ln (ecx + q(x)) ex mials p and q and positive numbers. k and c lim = ∞. x→∞ n! will be the fraction of degrees that is k . c Hence e dominates xn . x 54. lim ln x = lim xp = ∞. 59. If x → 0, then x2 → 0, so if lim f (x) = L, x→∞ x→∞ ln x ∞ x→0 g(x) lim is of type f (x2 ) x→∞ xp ∞ then lim = L (but not conversely). If we use L’Hˆpital’s Rule to get o x→0 g(x2 ) 1 x 1 f (x) lim p−1 = lim = 0 (since p > 0). a = 0 or 1, then lim involves the be- x→∞ px x→∞ pxp x→a g(x) p Therefore, x dominates ln x. f (x2 ) havior of the quotient near a, while lim t t x→a g(x2 ) 55. lim e 2 − t3 Since e 2 dominates t3 . So involves the behavior of the quotient near the t→∞ t different point a2 . lim e − t3 = ∞ 2 t→∞ 60. Functions f (x) = |x| and g(x) = x work. √ f (x) x − ln x ∞ lim does not exist as it approaches −1 56. lim √ is type . x→0 g(x) x→∞ x ∞ from the left and it approaches 1 from the we apply L’Hˆpital’s Rule to get o √ − 1 1 √ f (x2 ) 2 x x x−2 x right, but lim = 1. lim = lim x→0 g(x2 ) x→∞ 1 √ x→∞ x 2 x 2 2.5(4ωt − sin 4ωt) = lim 1 − √ = 1. 61. lim x→∞ x ω→0 4ω 2 2.5(4t − 4t cos 4ωt) = lim ln x3 + 2x + 1 ω→0 8ω 57. lim 2.5(16t2 sin 4ωt) x→∞ ln (x2 + x + 2) = lim =0 ω→0 8 we apply L’Hˆpital’s Rule o d dx ln x3 + 2x + 1 π lim 2.5 − 2.5 sin(4ωt + ) d 2 2 is type 0 ; dx (ln (x + x + 2)) x→∞ 62. lim 0 3x2 +2 ω→0 4ω 2 x3 +2x+1 we apply L’Hˆpital’s Rule to get o = lim 2x+1 −10t cos(4ωt + π ) x→∞ 2 x2 +x+2 lim 3x + 3x + 8x2 + 2x + 4 4 3 3 ω→0 8ω = lim = 40t2 sin(4ωt + π ) x→∞ 2x 4 + x3 + 4x2 + 4x + 1 2 = lim 2 = 5t2 . In general, for numerator and denominator the ω→0 8 highest degee of polynomials p and q, such that p(x) > 0 and q(x) > 0 for x > 0, 2 should be the lim ln(p(x)) . ln(q(x)) x→∞ 1.5 3x ln e + x ∞ 58. lim 2x + 4) is ; 1 x→∞ ln (e ∞ we apply L’Hˆpital’s Rule o 0.5 d 3x dx ln e +x lim d x→∞ dx (ln (e2x + 4)) 0 0 0.1 0.2 0.3 0.4 0.5 0.6 3e3x +1 t e3x +x = lim 2e2x x→∞ e2x +4 5x 3e + 12e3x + e2x + 4 3 63. The area of triangular region 1 is = lim = (1/2)(base)(height) x→∞ 2e5x + 2xe2x 2